Sei sulla pagina 1di 83

CALCULUS II

Solutions to Practice Problems


Paul Dawkins

Calculus II

Table of Contents
Preface............................................................................................................................................ iii Outline............................................................................................................................................ iii Integration Techniques .................................................................................................................. 6
Introduction ................................................................................................................................................ 6 Integration by Parts .................................................................................................................................... 6 Integrals Involving Trig Functions ........................................................................................................... 16 Trig Substitutions ..................................................................................................................................... 29 Partial Fractions ....................................................................................................................................... 52 Integrals Involving Roots ......................................................................................................................... 62 Integrals Involving Quadratics ................................................................................................................. 66 Integration Strategy .................................................................................................................................. 72 Improper Integrals .................................................................................................................................... 72 Comparison Test for Improper Integrals .................................................................................................. 72 Approximating Definite Integrals ............................................................................................................ 72

Applications of Integrals ............................................................................................................. 72


Introduction .............................................................................................................................................. 72 Arc Length ............................................................................................................................................... 73 Surface Area ............................................................................................................................................. 73 Center of Mass ......................................................................................................................................... 73 Hydrostatic Pressure and Force ................................................................................................................ 73 Probability ................................................................................................................................................ 73

Parametric Equations and Polar Coordinates .......................................................................... 73


Introduction .............................................................................................................................................. 74 Parametric Equations and Curves............................................................................................................. 74 Tangents with Parametric Equations ........................................................................................................ 74 Area with Parametric Equations ............................................................................................................... 74 Arc Length with Parametric Equations .................................................................................................... 74 Surface Area with Parametric Equations .................................................................................................. 75 Polar Coordinates ..................................................................................................................................... 75 Tangents with Polar Coordinates ............................................................................................................. 75 Area with Polar Coordinates .................................................................................................................... 75 Arc Length with Polar Coordinates .......................................................................................................... 75 Surface Area with Polar Coordinates ....................................................................................................... 75 Arc Length and Surface Area Revisited ................................................................................................... 75

Sequences and Series ................................................................................................................... 76


Introduction .............................................................................................................................................. 76 Sequences ................................................................................................................................................. 76 More on Sequences .................................................................................................................................. 76 Series The Basics .................................................................................................................................. 77 Series Convergence/Divergence............................................................................................................ 77 Series Special Series.............................................................................................................................. 77 Integral Test ............................................................................................................................................. 77 Comparison Test / Limit Comparison Test .............................................................................................. 77 Alternating Series Test ............................................................................................................................. 77 Absolute Convergence ............................................................................................................................. 77 Ratio Test ................................................................................................................................................. 77 Root Test .................................................................................................................................................. 78 Strategy for Series .................................................................................................................................... 78 Estimating the Value of a Series .............................................................................................................. 78 Power Series ............................................................................................................................................. 78 Power Series and Functions ..................................................................................................................... 78 Taylor Series ............................................................................................................................................ 78 Applications of Series .............................................................................................................................. 78 2007 Paul Dawkins i http://tutorial.math.lamar.edu/terms.aspx

Calculus II Binomial Series ........................................................................................................................................ 78

Vectors .......................................................................................................................................... 79
Introduction .............................................................................................................................................. 79 Vectors The Basics ................................................................................................................................ 79 Vector Arithmetic..................................................................................................................................... 79 Dot Product .............................................................................................................................................. 79 Cross Product ........................................................................................................................................... 79

Three Dimensional Space ............................................................................................................ 80


Introduction .............................................................................................................................................. 80 The 3-D Coordinate System ..................................................................................................................... 80 Equations of Lines.................................................................................................................................... 80 Equations of Planes .................................................................................................................................. 80 Quadric Surfaces ...................................................................................................................................... 81 Functions of Several Variables................................................................................................................. 81 Vector Functions ...................................................................................................................................... 81 Calculus with Vector Functions ............................................................................................................... 81 Tangent, Normal and Binormal Vectors .................................................................................................. 81 Arc Length with Vector Functions ........................................................................................................... 81 Curvature .................................................................................................................................................. 81 Velocity and Acceleration ........................................................................................................................ 82 Cylindrical Coordinates............................................................................................................................ 82 Spherical Coordinates .............................................................................................................................. 82

2007 Paul Dawkins

ii

http://tutorial.math.lamar.edu/terms.aspx

Calculus II

Preface
Here are the solutions to the practice problems for my Calculus II notes. Some solutions will have more or less detail than other solutions. As the difficulty level of the problems increases less detail will go into the basics of the solution under the assumption that if youve reached the level of working the harder problems then you will probably already understand the basics fairly well and wont need all the explanation. This document was written with presentation on the web in mind. On the web most solutions are broken down into steps and many of the steps have hints. Each hint on the web is given as a popup however in this document they are listed prior to each step. Also, on the web each step can be viewed individually by clicking on links while in this document they are all showing. Also, there are liable to be some formatting parts in this document intended for help in generating the web pages that havent been removed here. These issues may make the solutions a little difficult to follow at times, but they should still be readable.

Outline
Here is a list of sections for which problems have been written. Integration Techniques Integration by Parts No problems written yet. Integrals Involving Trig Functions No problems written yet. Trig Substitutions No problems written yet. Partial Fractions No problems written yet. Integrals Involving Roots No problems written yet. Integrals Involving Quadratics No problems written yet. Using Integral Tables No problems written yet. Integration Strategy No problems written yet. Improper Integrals No problems written yet. Comparison Test for Improper Integrals No problems written yet. Approximating Definite Integrals No problems written yet. Applications of Integrals Arc Length No problems written yet. Surface Area No problems written yet. Center of Mass No problems written yet. Hydrostatic Pressure and Force No problems written yet. Probability No problems written yet. Parametric Equations and Polar Coordinates Parametric Equations and Curves No problems written yet.
2007 Paul Dawkins iii http://tutorial.math.lamar.edu/terms.aspx

Calculus II

Tangents with Parametric Equations No problems written yet. Area with Parametric Equations No problems written yet. Arc Length with Parametric Equations No problems written yet. Surface Area with Parametric Equations No problems written yet. Polar Coordinates No problems written yet. Tangents with Polar Coordinates No problems written yet. Area with Polar Coordinates No problems written yet. Arc Length with Polar Coordinates No problems written yet. Surface Area with Polar Coordinates No problems written yet. Arc Length and Surface Area Revisited No problems written yet. Sequences and Series Sequences No problems written yet. More on Sequences No problems written yet. Series The Basics No problems written yet. Series Convergence/Divergence No problems written yet. Series Special Series No problems written yet. Integral Test No problems written yet. Comparison Test/Limit Comparison Test No problems written yet. Alternating Series Test No problems written yet. Absolute Convergence No problems written yet. Ratio Test No problems written yet. Root Test No problems written yet. Strategy for Series No problems written yet. Estimating the Value of a Series No problems written yet. Power Series No problems written yet. Power Series and Functions No problems written yet. Taylor Series No problems written yet. Applications of Series No problems written yet. Binomial Series No problems written yet. Vectors Vectors The Basics No problems written yet. Vector Arithmetic No problems written yet. Dot Product No problems written yet. Cross Product No problems written yet. Three Dimensional Space The 3-D Coordinate System No problems written yet. Equations of Lines No problems written yet. Equations of Planes No problems written yet. Quadric Surfaces No problems written yet. Functions of Several Variables No problems written yet. Vector Functions No problems written yet. Calculus with Vector Functions No problems written yet. Tangent, Normal and Binormal Vectors No problems written yet. Arc Length with Vector Functions No problems written yet. Curvature No problems written yet. Velocity and Acceleration No problems written yet. Cylindrical Coordinates No problems written yet. Spherical Coordinates No problems written yet.
2007 Paul Dawkins iv http://tutorial.math.lamar.edu/terms.aspx

Calculus II

2007 Paul Dawkins

http://tutorial.math.lamar.edu/terms.aspx

Calculus II

Integration Techniques
Introduction
Here are a set of problems for which no solutions are available. The main intent of these problems is to have a set of problems available for any instructors who are looking for some extra problems. Note that some sections will have more problems than others and some will have more or less of a variety of problems. Most sections should have a range of difficulty levels in the problems although this will vary from section to section. Here is a list of topics in this chapter that have problems written for them. Integration by Parts No problems written yet. Integrals Involving Trig Functions No problems written yet. Trig Substitutions No problems written yet. Partial Fractions No problems written yet. Integrals Involving Roots No problems written yet. Integrals Involving Quadratics No problems written yet. Using Integral Tables No problems written yet. Integration Strategy No problems written yet. Improper Integrals No problems written yet. Comparison Test for Improper Integrals No problems written yet. Approximating Definite Integrals No problems written yet.

Integration by Parts
1. Evaluate

4 x cos ( 2 - 3x ) dx

Hint : Remember that we want to pick u and dv so that upon computing du and v and plugging everything into the Integration by Parts formula the new integral is one that we can do. Step 1 The first step here is to pick u and dv. We want to choose u and dv so that when we compute du and v and plugging everything into the Integration by Parts formula the new integral we get is one that we can do. With that in mind it looks like the following choices for u and dv should work for us.

u = 4x

dv = cos ( 2 - 3x ) dx

2007 Paul Dawkins

http://tutorial.math.lamar.edu/terms.aspx

Calculus II

Step 2 Next we need to compute du (by differentiating u) and v (by integrating dv).

u = 4x dv = cos ( 2 - 3 x ) dx

du = 4dx v = - 1 sin ( 2 - 3 x ) 3

Step 3 Plugging u, du, v and dv into the Integration by Parts formula gives,

4 x cos ( 2 - 3x ) dx = ( 4 x ) ( -

1 3

sin ( 2 - 3x ) ) - - 4 sin ( 2 - 3x ) dx 3

= - 4 x sin ( 2 - 3 x ) + 4 sin ( 2 - 3 x ) dx 3 3
Step 4 Okay, the new integral we get is easily doable and so all we need to do to finish this problem out is do the integral.

4 x cos ( 2 - 3x ) dx =
( 2 + 5x ) e
6 0
1x 3

4 - 4 x sin ( 2 - 3 x ) + 9 cos ( 2 - 3 x ) + c 3

2. Evaluate

dx .

Hint : Remember that we want to pick u and dv so that upon computing du and v and plugging everything into the Integration by Parts formula the new integral is one that we can do. Also, dont forget that the limits on the integral wont have any effect on the choices of u and dv. Step 1 The first step here is to pick u and dv. We want to choose u and dv so that when we compute du and v and plugging everything into the Integration by Parts formula the new integral we get is one that we can do. With that in mind it looks like the following choices for u and dv should work for us.

u = 2 + 5x

dv = e 3 dx

1x

Step 2 Next we need to compute du (by differentiating u) and v (by integrating dv).

u = 2 + 5x dv = e 3 dx
Step 3
1x

du = 5dx v = 3e 3
1x

2007 Paul Dawkins

http://tutorial.math.lamar.edu/terms.aspx

Calculus II

We can deal with the limits as we do the integral or we can just do the indefinite integral and then take care of the limits in the last step. We will be using the later way of dealing with the limits for this problem. So, plugging u, du, v and dv into the Integration by Parts formula gives,

( 2 + 5x ) e

1x 3

= ( 2 + 5 x ) 3e 3 - 5 3e 3 dx = 3e 3 ( 2 + 5 x ) - 15 e 3 dx
1x 1x 1x 1x

( ) ( )
1x

Step 4 Okay, the new integral we get is easily doable so lets evaluate it to get,

( 2 + 5x ) e

1x 3

= 3e 3 ( 2 + 5 x ) - 45e 3 + c = 15 xe 3 - 39e 3 + c
1x 1x 1x

Step 5 The final step is then to take care of the limits.


x x x 6 ( 2 + 5x ) e dx = (15xe - 39e ) 0
1 3 1 3 1 3

0 6

= -39 - 15e 2 = -415.8419

Do not get excited about the fact that the lower limit is larger than the upper limit. This can happen on occasion and in no way affects how the integral is evaluated.

3. Evaluate

( 3t + t ) sin ( 2t ) dt
2

Hint : Remember that we want to pick u and dv so that upon computing du and v and plugging everything into the Integration by Parts formula the new integral is one that we can do (or at least will be easier to deal with). Step 1 The first step here is to pick u and dv. We want to choose u and dv so that when we compute du and v and plugging everything into the Integration by Parts formula the new integral we get is one that we can do, or will at least be an integral that will be easier to deal with. With that in mind it looks like the following choices for u and dv should work for us.

u = 3t + t 2

dv = sin ( 2t ) dt

Step 2 Next we need to compute du (by differentiating u) and v (by integrating dv).

u = 3t + t 2 dv = sin ( 2t ) dt

du = ( 3 + 2t ) dt v = - 1 cos ( 2t ) 2

2007 Paul Dawkins

http://tutorial.math.lamar.edu/terms.aspx

Calculus II

Step 3 Plugging u, du, v and dv into the Integration by Parts formula gives,

( 3t + t ) sin ( 2t ) dt = - ( 3t + t ) cos ( 2t ) + ( 3 + 2t ) cos ( 2t ) dt


2 1 2 2 1 2

Step 4 Now, the new integral is still not one that we can do with only Calculus I techniques. However, it is one that we can do another integration by parts on and because the power on the ts have gone down by one we are heading in the right direction. So, here are the choices for u and dv for the new integral.

u = 3 + 2t dv = cos ( 2t ) dt

du = 2dt v = 1 sin ( 2t ) 2

Step 5 Okay, all we need to do now is plug these new choices of u and dv into the new integral we got in Step 3 and finish the problem out.

( 3t + t ) sin ( 2t ) dt = - ( 3t + t ) cos ( 2t ) + ( 3 + 2t ) sin ( 2t ) - sin ( 2t ) dt = - ( 3t + t ) cos ( 2t ) + ( 3 + 2t ) sin ( 2t ) + cos ( 2t ) + c = - ( 3t + t ) cos ( 2t ) + ( 3 + 2t ) sin ( 2t ) + cos ( 2t ) + c
2 1 2 1 2 2 2 1 2 1 2 1 2 1 2 1 2 1 2 2 1 4 1 4

4. Evaluate

6 tan ( w ) dw .
-1 8

Hint : Be careful with your choices of u and dv here. If you think about it there is really only one way that the choice can be made here and have the problem be workable. Step 1 The first step here is to pick u and dv. Note that if we choose the inverse tangent for dv the only way to get v is to integrate dv and so we would need to know the answer to get the answer and so that wont work for us. Therefore, the only real choice for the inverse tangent is to let it be u. So, here are our choices for u and dv.
8 u = 6 tan -1 ( w )

dv = dw

Dont forget the dw! The differential dw still needs to be put into the dv even though there is nothing else left in the integral.

2007 Paul Dawkins

http://tutorial.math.lamar.edu/terms.aspx

Calculus II

Step 2 Next we need to compute du (by differentiating u) and v (by integrating dv).
8 u = 6 tan -1 ( w )

du = 6 v=w

8 1+ ( w )

8 - w2

dw = 6

8 - w2 64 1 + w2

dw

dv = dw

Step 3 In order to complete this problem well need to do some rewrite on du as follows,

du =

-48 dw w + 64
2

Plugging u, du, v and dv into the Integration by Parts formula gives,

6 tan ( w ) dw = 6w tan ( w ) + 48 w
-1 8 -1 8

w dw + 64

Step 4 Okay, the new integral we get is easily doable (with the substitution u = 64 + w2 ) and so all we need to do to finish this problem out is do the integral.

6 tan ( w ) dw =
-1 8

8 6 w tan -1 ( w ) + 24 ln w2 + 64 + c

5. Evaluate

2z

cos ( 1 z ) dz . 4

Hint : This is one of the few integration by parts problems where either function can go on u and dv. Be careful however to not get locked into an endless cycle of integration by parts. Step 1 The first step here is to pick u and dv. In this case we can put the exponential in either the u or the dv and the cosine in the other. It is one of the few problems where the choice doesnt really matter. For this problem well use the following choices for u and dv.

u = cos ( 1 z ) 4

dv = e2 z dz

Step 2 Next we need to compute du (by differentiating u) and v (by integrating dv).

2007 Paul Dawkins

10

http://tutorial.math.lamar.edu/terms.aspx

Calculus II

u = cos ( 1 z ) 4 dv = e 2 z dz

du = - 1 sin ( 1 z ) dz 4 4 v = 1 e2 z 2

Step 3 Plugging u, du, v and dv into the Integration by Parts formula gives,

2z

cos ( 1 z ) dz = 1 e2 z cos ( 1 z ) + 1 e2 z sin ( 1 z ) dz 4 2 4 8 4

Step 4 Well now need to do integration by parts again and to do this well use the following choices.

u = sin ( 1 z ) 4 dv = e 2 z dz

du = 1 cos ( 1 z ) dz 4 4

v = 1 e2 z 2

Step 5 Plugging these into the integral from Step 3 gives,

2z

cos ( 1 z ) dz = 1 e 2 z cos ( 1 z ) + 1 1 e 2 z sin ( 1 z ) - 1 e2 z cos ( 1 z ) dz 4 2 4 8 2 4 8 4


1 1 = 1 e 2 z cos ( 1 z ) + 16 e 2 z sin ( 1 z ) - 64 e 2 z cos ( 1 z ) dz 2 4 4 4

Step 6 To finish this problem all we need to do is some basic algebraic manipulation to get the identical integrals on the same side of the equal sign.
z e cos ( z ) dz = z e cos ( z ) dz = z e cos ( z ) dz =
2 1 4 2 2 1 4 1 4 1 2 1 2 1 2 1 1 e 2 z cos ( 1 z ) + 16 e 2 z sin ( 1 z ) - 64 e 2 z cos ( 1 z ) dz 4 4 4 1 e 2 z cos ( 1 z ) + 16 e 2 z sin ( 1 z ) 4 4 1 e 2 z cos ( 1 z ) + 16 e 2 z sin ( 1 z ) 4 4

2z

1 cos ( 1 z ) dz + 64 4 65 64

Finally, all we need to do is move the coefficient on the integral over to the right side.

2z

cos ( 1 z ) dz = 4

32 65

4 e 2 z cos ( 1 z ) + 65 e 2 z sin ( 1 z ) + c 4 4

6. Evaluate

x 2 cos ( 4 x ) dx .

Hint : Remember that we want to pick u and dv so that upon computing du and v and plugging everything into the Integration by Parts formula the new integral is one that we can do (or at least will be easier to deal with). Also, dont forget that the limits on the integral wont have any effect on the choices of u and dv. Step 1
2007 Paul Dawkins 11 http://tutorial.math.lamar.edu/terms.aspx

Calculus II

The first step here is to pick u and dv. We want to choose u and dv so that when we compute du and v and plugging everything into the Integration by Parts formula the new integral we get is one that we can do, or will at least be an integral that will be easier to deal with. With that in mind it looks like the following choices for u and dv should work for us.

u = x2

dv = cos ( 4 x ) dx

Step 2 Next we need to compute du (by differentiating u) and v (by integrating dv).

u = x2 dv = cos ( 4 x ) dx

du = 2 x dx v = 1 sin ( 4 x ) 4

Step 3 We can deal with the limits as we do the integral or we can just do the indefinite integral and then take care of the limits in the last step. We will be using the later way of dealing with the limits for this problem. So, plugging u, du, v and dv into the Integration by Parts formula gives,

cos ( 4 x ) dx = 1 x 2 sin ( 4 x ) - 1 x sin ( 4 x ) dx 4 2

Step 4 Now, the new integral is still not one that we can do with only Calculus I techniques. However, it is one that we can do another integration by parts on and because the power on the xs have gone down by one we are heading in the right direction. So, here are the choices for u and dv for the new integral.

u=x dv = sin ( 4 x ) dx

du = dx v = - 1 cos ( 4 x ) 4

Step 5 Okay, all we need to do now is plug these new choices of u and dv into the new integral we got in Step 3 and evaluate the integral.

cos ( 4 x ) dx = 1 x 2 sin ( 4 x ) - 1 - 1 x cos ( 4 x ) + 1 cos ( 4 x ) dx 4 2 4 4


1 = 1 x 2 sin ( 4 x ) + 1 x cos ( 4 x ) - 32 sin ( 4 x ) + c 4 8 1 = 1 x 2 sin ( 4 x ) - 1 - 1 x cos ( 4 x ) + 16 sin ( 4 x ) + c 4 2 4

Step 6 The final step is then to take care of the limits.

2007 Paul Dawkins

12

http://tutorial.math.lamar.edu/terms.aspx

Calculus II
1 x 2 cos ( 4 x ) dx = ( 1 x 2 sin ( 4 x ) + 1 x cos ( 4 x ) - 32 sin ( 4 x ) ) = 1 p 4 8 8

7. Evaluate

sin ( 2t 4 ) dt .

Hint : Be very careful with your choices of u and dv for this problem. It looks a lot like previous practice problems but it isnt! Step 1 The first step here is to pick u and dv and, in this case, well need to be careful how we chose them. If we follow the model of many of the examples/practice problems to this point it is tempting to let u be t 7 and to let dv be sin 2t 4 .

( )

However, this will lead to some real problems. To compute v wed have to integrate the sine and because of the t 4 in the argument this is not possible. In order to integrate the sine we would have to have a t 3 in the integrand as well in order to a substitution as shown below,

sin ( 2t 4 ) dt = 1 sin ( w ) dw = - 1 cos ( 2t 4 ) + c 8 8

w = 2t 4

Now, this may seem like a problem, but in fact its not a problem for this particular integral. Notice that we actually have 7 ts in the integral and there is no reason that we cant split them up as follows,

sin ( 2t 4 ) dt = t 4 t 3 sin ( 2t 4 ) dt

After doing this we can now choose u and dv as follows,

u = t4

dv = t 3 sin ( 2t 4 ) dt

Step 2 Next we need to compute du (by differentiating u) and v (by integrating dv).

u = t4 dv = t 3 sin ( 2t 4 ) dt

du = 4t 3 dt v = - 1 cos ( 2t 4 ) 8

Step 3 Plugging u, du, v and dv into the Integration by Parts formula gives,

t
2007 Paul Dawkins

sin ( 2t 4 ) dt = - 1 t 4 cos ( 2t 4 ) + 1 t 3 cos ( 2t 4 ) dt 8 2


13 http://tutorial.math.lamar.edu/terms.aspx

Calculus II

Step 4 At this point, notice that the new integral just requires the same Calculus I substitution that we used to find v. So, all we need to do is evaluate the new integral and well be done.

1 sin ( 2t 4 ) dt = - 1 t 4 cos ( 2t 4 ) + 16 sin ( 2t 4 ) + c 8

Do not get so locked into patterns for these problems that you end up turning the patterns into rules on how certain kinds of problems work. Most of the easily seen patterns are also easily broken (as this problem has shown). Because we (as instructors) tend to work a lot of easy problems initially they also tend to conform to the patterns that can be easily seen. This tends to lead students to the idea that the patterns will always work and then when they run into one where the pattern doesnt work they get in trouble. So, be careful! Note as well that were not saying that patterns dont exist and that it isnt useful to recognize them. You just need to be careful and understand that there will, on occasion, be problems where it will look like a pattern you recognize, but in fact will not quite fit the pattern and another approach will be needed to work the problem. Alternate Solution Note that there is an alternate solution to this problem. We could use the substitution w = 2t 4 as the first step as follows.

1 sin ( 2t 4 ) dt = t 4 t 3 sin ( 2t 4 ) dt = ( 1 w ) ( 1 ) sin ( w ) dw = 16 w sin ( w ) dw 2 8

w = 2t 4

dw = 8t 3 dt

&

t4 = 1 w 2

We wont avoid integration by parts as we can see here, but it is somewhat easier to see it this time. Here is the rest of the work for this problem.
1 u = 16 w

dv = sin ( w ) dw

v = - cos ( w )

1 du = 16 dw

1 1 1 1 sin ( 2t 4 ) dt = - 16 w cos ( w ) + 16 cos ( w ) dw = - 16 w cos ( w ) + 16 sin ( w ) + c

As the final step we just need to substitution back in for w.

t
8. Evaluate

1 sin ( 2t 4 ) dt = - 1 t 4 cos ( 2t 4 ) + 16 sin ( 2t 4 ) + c 8

cos ( 3 y ) dy .

Hint : Doing this with standard integration by parts would take a fair amount of time so maybe this would be a good candidate for the table method of integration by parts.
2007 Paul Dawkins 14 http://tutorial.math.lamar.edu/terms.aspx

Calculus II

Step 1 Okay, with this problem doing the standard method of integration by parts (i.e. picking u and dv and using the formula) would take quite a bit of time. So, this looks like a good problem to use the table that we saw in the notes to shorten the process up. Here is the table for this problem.

y6 6y
5 4 1 3

cos ( 3 y ) +
1 9

30 y 120 y 3 360 y 2 720 y

- cos ( 3 y ) + 1 - 27 sin ( 3 y ) 1 81 1 243

sin ( 3 y ) -

cos ( 3 y ) +

720 - cos ( 3 y ) + 1 0 - 2187 sin ( 3 y ) 1 729

sin ( 3 y ) -

Step 2 Heres the integral for this problem,

1 cos ( 3 y ) dy = ( y 6 ) ( 1 sin ( 3 y ) ) - ( 6 y 5 ) ( - 1 cos ( 3 y ) ) + ( 30 y 4 ) ( - 27 sin ( 3 y ) ) 3 9 1 1 - (120 y 3 ) ( 81 cos ( 3 y ) ) + ( 360 y 2 ) ( 243 sin ( 3 y ) ) 1 1 - ( 720 y ) ( - 729 cos ( 3 y ) ) + ( 720 ) ( - 2187 sin ( 3 y ) ) + c

1 3

40 y 6 sin ( 3 y ) + 2 y 5 cos ( 3 y ) - 10 y 4 sin ( 3 y ) - 27 y 3 cos ( 3 y ) 3 9 80 + 40 y 2 sin ( 3 y ) + 80 y cos ( 3 y ) - 243 sin ( 3 y ) + c 27 81

9. Evaluate

(4x

- 9 x 2 + 7 x + 3 ) e - x dx .

Hint : Doing this with standard integration by parts would take a fair amount of time so maybe this would be a good candidate for the table method of integration by parts. Step 1 Okay, with this problem doing the standard method of integration by parts (i.e. picking u and dv and using the formula) would take quite a bit of time. So, this looks like a good problem to use the table that we saw in the notes to shorten the process up. Here is the table for this problem.

2007 Paul Dawkins

15

http://tutorial.math.lamar.edu/terms.aspx

Calculus II

4 x 3 - 9 x 2 + 7 x + 3 e- x 12 x 2 - 18 x + 7 -e- x 24 x - 18 e- x 24 -e- x 0
Step 2 Heres the integral for this problem,

+ + +

e- x

(4x

- 9 x 2 + 7 x + 3 ) e - x dx = ( 4 x 3 - 9 x 2 + 7 x + 3)( -e- x ) - (12 x 2 - 18 x + 7 )( e - x ) + ( 24 x - 18 ) ( -e - x ) - ( 24 ) ( e - x ) + c = -e - x ( 4 x 3 - 9 x 2 + 7 x + 3) - e - x (12 x 2 - 18 x + 7 ) - e - x ( 24 x - 18 ) - 24e - x + c = -e - x ( 4 x 3 + 3 x 2 + 13 x + 16 )

Integrals Involving Trig Functions


1. Evaluate

sin ( x ) cos ( x ) dx
3 2 3 4 2 3

Hint : Pay attention to the exponents and recall that for most of these kinds of problems youll need to use trig identities to put the integral into a form that allows you to do the integral (usually with a Calc I substitution). Step 1 The first thing to notice here is that the exponent on the sine is odd and so we can strip one of them out.

sin ( x ) cos ( x ) dx = sin ( x ) cos ( x ) sin ( x ) dx


3 2 3 4 2 3 2 2 3 4 2 3 2 3

Step 2 Now we can use the trig identity sin 2 q + cos 2 q = 1 to convert the remaining sines to cosines.

sin ( x ) cos ( x ) dx = (1 - cos ( x ) ) cos ( x ) sin ( x ) dx


3 2 3 4 2 3 2 2 3 4 2 3 2 3

Step 3

2007 Paul Dawkins

16

http://tutorial.math.lamar.edu/terms.aspx

Calculus II

We can now use the substitution u = cos ( 2 x ) to evaluate the integral. 3

sin ( x ) cos ( x ) dx = - (1 - u ) u du = - u - u du = - (
3 2 3 4 2 3 3 2 3 2 2 4 4 6 3 2

1 5

u5 - 1 u 7 ) + c 7

Note that well not be doing the actual substitution work here. At this point it is assumed that you recall substitution well enough to fill in the details if you need to. If you are rusty on substitutions you should probably go back to the Calculus I practice problems and practice on the substitutions. Step 4 Dont forget to substitute back in for u!

sin ( x ) cos ( x ) dx =
3 2 3 4 2 3

3 14

3 cos7 ( 2 x ) - 10 cos5 ( 2 x ) + c 3 3

2. Evaluate

sin ( 3z ) cos ( 3 z ) dz
8 5

Hint : Pay attention to the exponents and recall that for most of these kinds of problems youll need to use trig identities to put the integral into a form that allows you to do the integral (usually with a Calc I substitution). Step 1 The first thing to notice here is that the exponent on the cosine is odd and so we can strip one of them out.

sin ( 3z ) cos ( 3z ) dz = sin ( 3 z ) cos ( 3z ) cos ( 3 z ) dz


8 5 8 4

Step 2 Now we can use the trig identity sin 2 q + cos 2 q = 1 to convert the remaining cosines to sines.

sin ( 3z ) cos ( 3z ) dz = sin ( 3z ) cos ( 3 z ) cos ( 3z ) dz = sin ( 3 z ) 1 - sin ( 3 z ) cos ( 3 z ) dz


8 5 8 2 2 8 2 2

Step 3 We can now use the substitution u = sin ( 3 z ) to evaluate the integral.

sin ( 3z ) cos ( 3z ) dz = u = u
8 5 1 3 1 3

8 8

1 - u 2 du

2 1 - 2u10 + u12 du = 1 ( 1 u 9 - 11 u11 + 13 u13 ) + c 3 9

2007 Paul Dawkins

17

http://tutorial.math.lamar.edu/terms.aspx

Calculus II

Note that well not be doing the actual substitution work here. At this point it is assumed that you recall substitution well enough to fill in the details if you need to. If you are rusty on substitutions you should probably go back to the Calculus I practice problems and practice on the substitutions. Step 4 Dont forget to substitute back in for u!

sin ( 3z ) cos ( 3z ) dz =
8 5

1 27

2 1 sin 9 ( 3z ) - 33 sin11 ( 3z ) + 39 sin13 ( 3z ) + c

3. Evaluate

cos ( 2t ) dt
4

Hint : Pay attention to the exponents and recall that for most of these kinds of problems youll need to use trig identities to put the integral into a form that allows you to do the integral (usually with a Calc I substitution). Step 1 The first thing to notice here is that we only have even exponents and so well need to use halfangle and double-angle formulas to reduce this integral into one that we can do. Also, do not get excited about the fact that we dont have any sines in the integrand. Sometimes we will not have both trig functions in the integrand. That doesnt mean that that we cant use the same techniques that we used in this section. So, lets start this problem off as follows.

cos ( 2t ) dt = ( cos ( 2t ) )
4 2

dt

Step 2 Now we can use the half-angle formula to get,

cos ( 2t ) dt = (1 + cos ( 4t ) )
4 1 2

dt = 1 (1 + 2 cos ( 4t ) + cos 2 ( 4t ) ) dt 4

Step 3 Well need to use the half-angle formula one more time on the third term to get,

cos ( 2t ) dt = 1 + 2 cos ( 4t ) + = + 2 cos ( 4t ) +


4 1 4 1 4 3 2

1 2 1 2

1 + cos ( 8t ) dt cos ( 8t ) dt

Step 4 Now all we have to do is evaluate the integral.

2007 Paul Dawkins

18

http://tutorial.math.lamar.edu/terms.aspx

Calculus II

cos ( 2t ) dt = (
4 1 4

3 2

1 3 1 t + 1 sin ( 4t ) + 16 sin ( 8t ) ) + c = 8 t + 1 sin ( 4t ) + 64 sin ( 8t ) + c 2 8

4. Evaluate

2p

cos3 ( 1 w ) sin 5 ( 1 w ) dw . 2 2

Hint : Pay attention to the exponents and recall that for most of these kinds of problems youll need to use trig identities to put the integral into a form that allows you to do the integral (usually with a Calc I substitution). Step 1 We have two options for dealing with the limits. We can deal with the limits as we do the integral or we can evaluate the indefinite integral and take care of the limits in the last step. Well use the latter method of dealing with the limits for this problem. In this case notice that both exponents are odd. This means that we can either strip out a cosine and convert the rest to sines or strip out a sine and convert the rest to cosines. Either are perfectly acceptable solutions. However, the exponent on the cosine is smaller and so there will be less conversion work if we strip out a cosine and convert the remaining cosines to sines. Here is that work.

cos ( w ) sin ( w ) dw = cos ( w ) sin ( w ) cos ( w ) dw = (1 - sin ( w ) ) sin ( w ) cos ( w ) dw


3 1 2 5 1 2 2 1 2 5 1 2 1 2 2 1 2 5 1 2 1 2

Step 2 We can now use the substitution u = sin ( 1 w ) to evaluate the integral. 2

cos ( w) sin ( w ) dw = 2 (1 - u ) u du = 2 u - u du = 2 (
3 1 2 5 1 2 2 5 5 7

1 6

u 6 - 1 u8 ) + c 8

Note that well not be doing the actual substitution work here. At this point it is assumed that you recall substitution well enough to fill in the details if you need to. If you are rusty on substitutions you should probably go back to the Calculus I practice problems and practice on the substitutions.
Step 3 Dont forget to substitute back in for u!

cos ( w) sin ( w) dw =
3 1 2 5 1 2

1 3

sin 6 ( 1 w) - 1 sin8 ( 1 w) + c 2 4 2

Step 4 Now all we need to do is deal with the limits.


2007 Paul Dawkins 19 http://tutorial.math.lamar.edu/terms.aspx

Calculus II

2p

cos3 ( 1 w) sin 5 ( 1 w) dw = ( 1 sin 6 ( 1 w) - 1 sin8 ( 1 w) ) 2 2 3 2 4 2

2p

1 = - 12

Alternate Solution As we noted above we could just have easily stripped out a sine and converted the rest to cosines if wed wanted to. Well not put that work in here, but here is the indefinite integral that you should have gotten had you done it that way.

cos ( w ) sin ( w ) dw = 3 1 2 5 1 2

1 2

cos 4 ( 1 w ) + 2 cos 6 ( 1 w ) - 1 cos8 ( 1 w ) + c 2 3 2 4 2

Note as well that regardless of which approach we use to doing the indefinite integral the value of the definite integral will be the same.

5. Evaluate

sec ( 3 y ) tan ( 3 y ) dy
6 2

Hint : Pay attention to the exponents and recall that for most of these kinds of problems youll need to use trig identities to put the integral into a form that allows you to do the integral (usually with a Calc I substitution). Step 1 The first thing to notice here is that the exponent on the secant is even and so we can strip two of them out.

sec ( 3 y ) tan ( 3 y ) dy = sec ( 3 y ) tan ( 3 y ) sec ( 3 y ) dy


6 2 4 2 2

Step 2 Now we can use the trig identity tan 2 q + 1 = sec 2 q to convert the remaining secants to tangents.

sec ( 3 y ) tan ( 3 y ) dy = sec ( 3 y ) tan ( 3 y ) sec ( 3 y ) dy = tan ( 3 y ) + 1 tan ( 3 y ) sec ( 3 y ) dy


6 2 2 2 2 2 2 2 2 2

Step 3 We can now use the substitution u = tan ( 3 y ) to evaluate the integral.

sec ( 3 y ) tan ( 3 y ) dy = u + 1 u du = u + 2u + u du = (
6 2 1 3 1 3 2 2 2 6 4 2 1 3

1 7

u 7 + 2 u5 + 1 u3 ) + c 5 3

Note that well not be doing the actual substitution work here. At this point it is assumed that you recall substitution well enough to fill in the details if you need to. If you are rusty on substitutions you should probably go back to the Calculus I practice problems and practice on the substitutions. Step 4
2007 Paul Dawkins 20 http://tutorial.math.lamar.edu/terms.aspx

Calculus II

Dont forget to substitute back in for u!

sec ( 3 y ) tan ( 3 y ) dy =
6 2

1 21

2 tan 7 ( 3 y ) + 15 tan 5 ( 3 y ) + 1 tan 3 ( 3 y ) + c 9

6. Evaluate

tan ( 6 x ) sec ( 6 x ) dx
3 10

Hint : Pay attention to the exponents and recall that for most of these kinds of problems youll need to use trig identities to put the integral into a form that allows you to do the integral (usually with a Calc I substitution). Step 1 The first thing to notice here is that the exponent on the tangent is odd and weve got a secant in the problems and so we can strip one of each of them out.

tan ( 6 x ) sec ( 6 x ) dx = tan ( 6 x ) sec ( 6 x )


3 10 2 9

tan ( 6 x ) sec ( 6 x ) dx

Step 2 Now we can use the trig identity tan 2 q + 1 = sec 2 q to convert the remaining tangents to secants.

tan ( 6 x ) sec ( 6 x ) dx = sec ( 6 x ) - 1 sec ( 6 x )


3 10 2 9

tan ( 6 x ) sec ( 6 x ) dx

Note that because the exponent on the secant is even we could also have just stripped two of them out and converted the rest of them to tangents. However, that conversion process would have been significantly more work than the path that we chose here. Step 3 We can now use the substitution u = sec ( 6 x ) to evaluate the integral.

tan ( 6 x ) sec ( 6 x ) dx = u - 1 u du = u - u du = (
3 10 1 6 1 6 2 9 11 9 1 6

1 12

1 u12 - 10 u10 ) + c

Note that well not be doing the actual substitution work here. At this point it is assumed that you recall substitution well enough to fill in the details if you need to. If you are rusty on substitutions you should probably go back to the Calculus I practice problems and practice on the substitutions. Step 4 Dont forget to substitute back in for u!

tan ( 6 x ) sec ( 6 x ) dx =
3 10

1 72

1 sec12 ( 6 x ) - 60 sec10 ( 6 x ) + c

2007 Paul Dawkins

21

http://tutorial.math.lamar.edu/terms.aspx

Calculus II

7. Evaluate

p 4

tan 7 ( z ) sec3 ( z ) dz .

Hint : Pay attention to the exponents and recall that for most of these kinds of problems youll need to use trig identities to put the integral into a form that allows you to do the integral (usually with a Calc I substitution). Step 1 We have two options for dealing with the limits. We can deal with the limits as we do the integral or we can evaluate the indefinite integral and take care of the limits in the last step. Well use the latter method of dealing with the limits for this problem. The first thing to notice here is that the exponent on the tangent is odd and weve got a secant in the problems and so we can strip one of each of them out and use the trig identity tan 2 q + 1 = sec 2 q to convert the remaining tangents to secants.

tan ( z ) sec ( z ) dz = tan ( z ) sec ( z ) tan ( z ) sec ( z ) dz = tan ( z ) sec ( z ) tan ( z ) sec ( z ) dz = sec ( z ) - 1 sec ( z ) tan ( z ) sec ( z ) dz
7 3 6 2 2 3 2 2 3 2

Step 2 We can now use the substitution u = sec ( z ) to evaluate the integral.

tan ( z ) sec ( z ) dz = u - 1 u du = u - 3u + 3u - u
7 3 2 3 2 8 6 4

3 3 du = 1 u 9 - 7 u 7 + 5 u 5 - 1 u 3 + c 9 3

Note that well not be doing the actual substitution work here. At this point it is assumed that you recall substitution well enough to fill in the details if you need to. If you are rusty on substitutions you should probably go back to the Calculus I practice problems and practice on the substitutions.
Step 3 Dont forget to substitute back in for u!

tan ( z ) sec ( z ) dz =
7 3

1 9

3 sec9 ( z ) - 7 sec7 ( z ) + 3 sec5 ( z ) - 1 sec3 ( z ) + c 5 3

Step 4 Now all we need to do is deal with the limits.

2007 Paul Dawkins

22

http://tutorial.math.lamar.edu/terms.aspx

Calculus II

p 4

3 tan 7 ( z ) sec3 ( z ) dz = ( 1 sec9 ( z ) - 7 sec7 ( z ) + 3 sec5 ( z ) - 1 sec3 ( z ) ) 9 5 3

p 4

2 315

(8 + 13 2 ) = 0.1675

8. Evaluate

cos ( 3t ) sin (8t ) dt

Step 1 There really isnt all that much to this problem. All we have to do is use the formula given in this section for reducing a product of a sine and a cosine into a sum. Doing this gives,

cos ( 3t ) sin (8t ) dt =

1 2

sin ( 8t - 3t ) + sin ( 8t + 3t ) dt =

1 2

sin ( 5t ) + sin (11t ) dt

Make sure that you pay attention to the formula! The formula given in this section listed the sine first instead of the cosine. Make sure that you used the formula correctly! Step 2 Now all we need to do is evaluate the integral.

cos ( 3t ) sin (8t ) dt = ( 1 2

1 5

1 1 1 cos ( 5t ) - 11 cos (11t ) ) + c = - 10 cos ( 5t ) - 22 cos (11t ) + c

9. Evaluate

sin ( 8 x ) sin ( x ) dx .

Step 1 There really isnt all that much to this problem. All we have to do is use the formula given in this section for reducing a product of a sine and a cosine into a sum. Doing this gives,

sin ( 8 x ) sin ( x ) dx =

1 1 2

cos ( 8 x - x ) - cos ( 8 x + x ) dx =

3 1 2 1

cos ( 7 x ) - cos ( 9 x ) dx

Step 2 Now all we need to do is evaluate the integral.

sin ( 8 x ) sin ( x ) dx = 1 1 sin ( 7 x ) - 1 sin ( 9 x ) 2 7 9 =


1 14

3 1

1 1 1 sin (12 ) - 18 sin ( 27 ) - 14 sin ( 7 ) + 18 sin ( 9 ) = -0.0174

Make sure your calculator is set to radians if you computed a decimal answer!

2007 Paul Dawkins

23

http://tutorial.math.lamar.edu/terms.aspx

Calculus II

10. Evaluate

cot (10 z ) csc (10 z ) dz


4

Hint : Even though no examples of products of cotangents and cosecants were done in the notes for this section you should know how to do them. Ask yourself how you would do the problem if it involved tangents and secants instead and you should be able to see how to do this problem as well. Step 1 Other than the obvious difference in the actual functions there is no practical difference in how this problem and one that had tangents and secants would work. So, all we need to do is ask ourselves how this would work if it involved tangents and secants and well be able to work this on as well. We can first notice here is that the exponent on the cotangent is odd and weve got a cosecant in the problems and so we can strip the (only) cotangent and one of the secants out.

cot (10 z ) csc (10 z ) dz = csc (10 z )


4 3

cot (10 z ) csc (10 z ) dz

Step 2 Normally we would use the trig identity cot 2 q + 1 = csc 2 q to convert the remaining cotangents to cosecants. However, in this case there are no remaining cotangents to convert and so there really isnt anything to do at this point other than to use the substitution u = csc (10 z ) to evaluate the integral.

cot (10 z ) csc (10z ) dz = - u


4 1 10

1 du = - 40 u 4 + c

Note that well not be doing the actual substitution work here. At this point it is assumed that you recall substitution well enough to fill in the details if you need to. If you are rusty on substitutions you should probably go back to the Calculus I practice problems and practice on the substitutions. Step 3 Dont forget to substitute back in for u!

cot (10 z ) csc (10 z ) dz = 4

1 40

csc4 (10 z ) + c

11. Evaluate

csc ( w ) cot ( w ) dw
6 1 4 4 1 4

Hint : Even though no examples of products of cotangents and cosecants were done in the notes for this section you should know how to do them. Ask yourself how you would do the problem if it involved tangents and secants instead and you should be able to see how to do this problem as well.
2007 Paul Dawkins 24 http://tutorial.math.lamar.edu/terms.aspx

Calculus II

Step 1 Other than the obvious difference in the actual functions there is no practical difference in how this problem and one that had tangents and secants would work. So, all we need to do is ask ourselves how this would work if it involved tangents and secants and well be able to work this on as well. We can first notice here is that the exponent on the cosecant is even and so we can strip out two of them.

csc ( w ) cot ( w ) dw = csc ( w ) cot ( w )


6 1 4 4 1 4 4 1 4 4 1 4

csc 2 ( 1 w ) dw 4

Step 2 Now we can use the trig identity cot 2 q + 1 = csc 2 q to convert the remaining cosecants to cotangents.

csc ( w) cot ( w) dw = csc ( w) cot ( w) csc ( w) dw = cot ( w ) + 1 cot ( w ) csc ( w ) dw


6 1 4 4 1 4 2 1 4 2 4 1 4 2 1 4 2 1 4 2 4 1 4 2 1 4

Step 3 Now we can use the substitution u = cot ( 1 w ) to evaluate the integral. 4

csc ( w ) cot ( w) dw = -4 u + 1 u du = -4 u + 2u + u du = -4 (
6 1 4 4 1 4 2 2 4 8 6 4

1 9

2 u9 + 7 u 7 + 1 u5 ) + c 5

Note that well not be doing the actual substitution work here. At this point it is assumed that you recall substitution well enough to fill in the details if you need to. If you are rusty on substitutions you should probably go back to the Calculus I practice problems and practice on the substitutions. Step 4 Dont forget to substitute back in for u!

csc ( w) cot ( w) dw = 6 1 4 4 1 4

4 9

4 cot 9 ( 1 w) - 8 cot 7 ( 1 w) - 5 cot 5 ( 1 w) + c 4 7 4 4

sec 4 ( 2t ) 12. Evaluate dt . 9 tan ( 2t )


Hint : How would you do this problem if it were a product? Step 1

2007 Paul Dawkins

25

http://tutorial.math.lamar.edu/terms.aspx

Calculus II

If this were a product of secants and tangents we would know how to do it. The same ideas work here, except that we have to pay attention to only the numerator. We cant strip anything out of the denominator (in general) and expect it to work the same way. We can only strip things out of the numerator. So, lets notice here is that the exponent on the secant is even and so we can strip out two of them.

sec 4 ( 2t ) sec 2 ( 2t ) 2 dt = sec ( 2t ) dt tan 9 2t 9 ( ) tan ( 2t )


Step 2 Now we can use the trig identity tan 2 q + 1 = sec 2 q to convert the remaining secants to tangents.

sec4 ( 2t ) tan 2 ( 2t ) + 1 2 sec ( 2t ) dt dt = tan 9 2t 9 ( ) tan ( 2t )


Step 3 Now we can use the substitution u = tan ( 2t ) to evaluate the integral.

sec 4 ( 2t ) u2 +1 dt = 1 9 du = 2 tan 9 2t u ( )

1 2

-7

+ u -9 du = 1 - 1 u -6 - 1 u -8 + c 2 6 8

Note that well not be doing the actual substitution work here. At this point it is assumed that you recall substitution well enough to fill in the details if you need to. If you are rusty on substitutions you should probably go back to the Calculus I practice problems and practice on the substitutions. Step 4 Dont forget to substitute back in for u!

sec4 ( 2t ) 1 tan 9 2t dt = - 12 ( )

1 tan 6 ( 2 t )

1 1 1 - 16 tan81( 2t ) + c = - 12 cot 6 ( 2t ) - 16 cot 8 ( 2t ) + c

13. Evaluate

2 + 7 sin 3 ( z ) dz . 2 cos ( z )

Hint : How would you do this problem if it were a product? Step 1 Because of the sum in the numerator it makes some sense (hopefully) to maybe split the integrand (and then the integral) up into two as follows.
2007 Paul Dawkins 26 http://tutorial.math.lamar.edu/terms.aspx

Calculus II

7 sin 3 ( z ) 2 + 7 sin 3 ( z ) 7 sin 3 ( z ) 2 2 dz = + dz = dz + dz cos 2 z 2 2 2 2 ( ) cos ( z ) cos ( z ) cos ( z ) cos ( z )


Step 2 Now, the first integral looks difficult at first glance, but we can easily rewrite this in terms of secants at which point it becomes a really easy integral. For the second integral again, think about how we would do that if it was a product instead of a quotient. In that case we would simply strip out a sine.

2 + 7 sin 3 ( z ) sin 2 ( z ) 2 sin ( z ) dz dz = 2sec ( z ) dz + 7 cos 2 z 2 ( ) cos ( z )


Step 3 As noted above the first integral is now very easy (which well do in the next step) and for the second integral we can use the trig identity sin 2 q + cos 2 q = 1 to convert the remaining sines in the second integral to cosines.

2 + 7 sin 3 ( z ) 1 - cos 2 ( z ) dz = 2sec2 ( z ) dz + 7 sin ( z ) dz cos 2 z 2 ( ) cos ( z )


Step 4 Now we can use the substitution u = cos ( z ) to evaluate the second integral. The first integral doesnt need any extra work.

2 + 7 sin 3 ( z ) 1- u2 dz = 2 tan ( z ) - 7 2 du cos 2 z u ( )

= 2 tan ( z ) - 7 u -2 - 1 du = 2 tan ( z ) - 7 ( -u -1 - u ) + c

Note that well not be doing the actual substitution work here. At this point it is assumed that you recall substitution well enough to fill in the details if you need to. If you are rusty on substitutions you should probably go back to the Calculus I practice problems and practice on the substitutions. Step 5 Dont forget to substitute back in for u!

2 + 7sin 3 ( z ) dz = 2 tan ( z ) + 7 cos1( z ) + 7 cos ( z ) + c = 2 tan ( z ) + 7 sec ( z ) + 7 cos ( z ) + c cos2 z ( )

14. Evaluate

9 sin ( 3x ) - 2 cos ( 3 x ) csc ( 3 x ) dx


5 3 4

.
http://tutorial.math.lamar.edu/terms.aspx

2007 Paul Dawkins

27

Calculus II

Hint : Since this has a mix of trig functions maybe the best option would be to first get it reduced down to just a couple that we know how to deal with. Step 1 To get started on this problem we should first probably see if we can reduce the integrand down to just sines and cosines. This is easy enough to do simply by recalling the definition of cosecant in terms of sine.

1 9sin ( 3x ) - 2 cos ( 3x ) csc ( 3x ) dx = 9sin ( 3x ) - 2 cos ( 3x ) sin ( 3x ) dx


5 3 4 5 3 4

cos3 ( 3x ) = 9sin ( 3 x ) - 2 4 dx sin ( 3x )


Step 2 The first integral is simple enough to do without any extra work. For the second integral again, think about how we would do that if it was a product instead of a quotient. In that case we would simply strip out a cosine.

cos 2 ( 3 x ) 9sin ( 3x ) - 2 cos ( 3x ) csc ( 3x ) dx = 9sin ( 3x ) - 2 sin 4 ( 3x ) cos ( 3x ) dx


5 3 4

Step 3 For the second integral we can use the trig identity sin 2 q + cos 2 q = 1 to convert the remaining cosines to sines.

1 - sin 2 ( 3x ) 9sin ( 3x ) - 2 cos ( 3x ) csc ( 3x ) dx = 9sin ( 3x ) dx - 2 sin 4 ( 3x ) cos ( 3x ) dx


5 3 4

Step 4 Now we can use the substitution u = sin ( 3x ) to evaluate the second integral. The first integral doesnt need any extra work.

1- u 5 3 4 9sin ( 3x ) - 2 cos ( 3x ) csc ( 3x ) dx = 9sin ( 3x ) dx - 23 u 4 du


2

= -3cos ( 3 x ) - 2 ( - 1 u -3 + u -1 ) + c 3 3
Note that well not be doing the actual substitution work here. At this point it is assumed that you recall substitution well enough to fill in the details if you need to. If you are rusty on substitutions you should probably go back to the Calculus I practice problems and practice on the substitutions.
2007 Paul Dawkins 28 http://tutorial.math.lamar.edu/terms.aspx

= 9sin ( 3x ) dx - 2 u -4 - u -2 du 3

Calculus II

Step 5 Dont forget to substitute back in for u!

9sin ( 3x ) - 2cos ( 3x ) csc ( 3x ) dx = -3cos ( 3x ) +


5 3 4

2 1 9 sin3 ( 3 x )

- 2 sin (13 x ) + c 3

2 = -3cos ( 3x ) + 9 csc3 ( 3x ) - 2 csc ( 3x ) + c 3

Trig Substitutions
1. Use a trig substitution to eliminate the root in

4 - 9z 2 .

Hint : When determining which trig function to use for the substitution recall from the notes in this section that we will use one of three trig identities to convert the sum or difference under the root into a single trig function. Which trig identity is closest to the quantity under the root? Step 1 The first step is to figure out which trig function to use for the substitution. To determine this notice that (ignoring the numbers) the quantity under the root looks similar to the identity,

1 - sin 2 (q ) = cos 2 (q )

So, it looks like sine is probably the correct trig function to use for the substitution. Now, we need to deal with the numbers on the two terms. Hint : In order to actually use the identity from the first step we need to get the numbers in each term to be identical upon doing the substitution. So, what would the coefficient of the trig function need to be in order to convert the coefficient of the variable into the constant term once weve done the substitution? Step 2 To get the coefficient on the trig function notice that we need to turn the 9 into a 4 once weve substituted the trig function in for z and squared the substitution out. With that in mind it looks like the substitution should be,

z = 2 sin (q ) 3
Now, all we have to do is actually perform the substitution and eliminate the root. Step 3

2007 Paul Dawkins

29

http://tutorial.math.lamar.edu/terms.aspx

Calculus II

4 4 - 9 z 2 = 4 - 9 ( 2 sin (q ) ) = 4 - 9 ( 9 ) sin 2 (q ) 3 2

= 4 - 4sin 2 (q ) = 2 1 - sin 2 (q ) = 2 cos 2 (q ) = 2 cos (q )


Note that because we dont know the values of q we cant determine if the cosine is positive or negative and so cannot get rid of the absolute value bars here.

2. Use a trig substitution to eliminate the root in

13 + 25x 2 .

Hint : When determining which trig function to use for the substitution recall from the notes in this section that we will use one of three trig identities to convert the sum or difference under the root into a single trig function. Which trig identity is closest to the quantity under the root? Step 1 The first step is to figure out which trig function to use for the substitution. To determine this notice that (ignoring the numbers) the quantity under the root looks similar to the identity,

1 + tan 2 (q ) = sec 2 (q )

So, it looks like tangent is probably the correct trig function to use for the substitution. Now, we need to deal with the numbers on the two terms. Hint : In order to actually use the identity from the first step we need to get the numbers in each term to be identical upon doing the substitution. So, what would the coefficient of the trig function need to be in order to convert the coefficient of the variable into the constant term once weve done the substitution? Step 2 To get the coefficient on the trig function notice that we need to turn the 25 into a 13 once weve substituted the trig function in for x and squared the substitution out. With that in mind it looks like the substitution should be,

x=

13 5

tan (q )

Now, all we have to do is actually perform the substitution and eliminate the root. Step 3

13 + 25 x 2 = 13 + 25

13 5

tan (q )

= 13 + 25 ( 13 ) tan 2 (q ) 25

= 13 + 13 tan 2 (q ) = 13 1 + tan 2 (q ) = 13 sec 2 (q ) = 13 sec (q )

2007 Paul Dawkins

30

http://tutorial.math.lamar.edu/terms.aspx

Calculus II

Note that because we dont know the values of q we cant determine if the secant is positive or negative and so cannot get rid of the absolute value bars here.

3. Use a trig substitution to eliminate the root in 7t 2 - 3

5 2

Hint : When determining which trig function to use for the substitution recall from the notes in this section that we will use one of three trig identities to convert the sum or difference under the root into a single trig function. Which trig identity is closest to the quantity under the root? Step 1 First, notice that there really is a root here as the term can be written as,

( 7t 2 - 3 ) 2 = ( 7t 2 - 3 ) 2 = 7t 2 - 3
5 1

Now, we need to figure out which trig function to use for the substitution. To determine this notice that (ignoring the numbers) the quantity under the root looks similar to the identity,

sec 2 (q ) - 1 = tan 2 (q )

So, it looks like secant is probably the correct trig function to use for the substitution. Now, we need to deal with the numbers on the two terms. Hint : In order to actually use the identity from the first step we need to get the numbers in each term to be identical upon doing the substitution. So, what would the coefficient of the trig function need to be in order to convert the coefficient of the variable into the constant term once weve done the substitution? Step 2 To get the coefficient on the trig function notice that we need to turn the 7 into a 3 once weve substituted the trig function in for t and squared the substitution out. With that in mind it looks like the substitution should be,

t=

3 7

sec (q )

Now, all we have to do is actually perform the substitution and eliminate the root. Step 3

2007 Paul Dawkins

31

http://tutorial.math.lamar.edu/terms.aspx

Calculus II

( 7t 2 - 3) 2 = 7t 2 - 3
5

= 7

3 7

sec (q )

5 3 - 3 = 7 ( 7 ) sec 2 (q ) - 3 5 5

= 3sec 2 (q ) - 3 = 3 sec2 (q ) - 1
5 5 = 3 tan 2 (q ) = 3 2 tan (q )

Note that because we dont know the values of q we cant determine if the tangent is positive or negative and so cannot get rid of the absolute value bars here.

4. Use a trig substitution to eliminate the root in

( w + 3)

- 100 .

Hint : Just because this looks a little different from the first couple of problems in this section doesnt mean that it works any differently. The term under the root still looks vaguely like one of three trig identities we need to use to convert the quantity under the root into a single trig function. Step 1 Okay, first off we need to acknowledge that this does look a little bit different from the first few problems in this section. However, it isnt really all that different. We still have a difference between a squared term with a variable in it and a number. This looks similar to the following trig identity (ignoring the coefficients as usual).

sec 2 (q ) - 1 = tan 2 (q )
So, secant is the trig function well need to use for the substitution here and we now need to deal with the numbers on the terms and get the substitution set up. Hint : Dealing with the numbers in this case is no different than the first few problems in this section. Step 2 Before dealing with the coefficient on the trig function lets notice that well be substituting in for w + 3 in this case since that is the quantity that is being squared in the first term. So, to get the coefficient on the trig function notice that we need to turn the 1 (i.e. the coefficient of the squared term) into a 100 once weve done the substitution. With that in mind it looks like the substitution should be,

w + 3 = 10 sec (q )
Now, all we have to do is actually perform the substitution and eliminate the root.
2007 Paul Dawkins 32 http://tutorial.math.lamar.edu/terms.aspx

Calculus II

Step 3

( w + 3)

- 100 =

(10sec (q ) )

- 100 = 100sec2 (q ) - 100 = 10 sec2 (q ) - 1

= 10 tan 2 (q ) = 10 tan (q )
Note that because we dont know the values of q we cant determine if the tangent is positive or negative and so cannot get rid of the absolute value bars here.

5. Use a trig substitution to eliminate the root in

4 ( 9t - 5 ) + 1 .
2

Hint : Just because this looks a little different from the first couple of problems in this section doesnt mean that it works any differently. The term under the root still looks vaguely like one of three trig identities we need to use to convert the quantity under the root into a single trig function. Step 1 Okay, first off we need to acknowledge that this does look a little bit different from the first few problems in this section. However, it isnt really all that different. We still have a sum of a squared term with a variable in it and a number. This looks similar to the following trig identity (ignoring the coefficients as usual).

tan 2 (q ) + 1 = sec 2 (q )
So, tangent is the trig function well need to use for the substitution here and we now need to deal with the numbers on the terms and get the substitution set up. Hint : Dealing with the numbers in this case is no different than the first few problems in this section. Step 2 Before dealing with the coefficient on the trig function lets notice that well be substituting in for 9t - 5 in this case since that is the quantity that is being squared in the first term. So, to get the coefficient on the trig function notice that we need to turn the 4 (i.e. the coefficient of the squared term) into a 1 once weve done the substitution. With that in mind it looks like the substitution should be,

9t - 5 = 1 tan (q ) 2
Now, all we have to do is actually perform the substitution and eliminate the root. Step 3

2007 Paul Dawkins

33

http://tutorial.math.lamar.edu/terms.aspx

Calculus II

4 ( 9t - 5 ) + 1 = 4 ( 1 tan (q ) ) + 1 = 4 ( 1 ) tan 2 (q ) + 1 = tan 2 (q ) + 1 2 4


2 2

= sec 2 (q ) = sec (q )
Note that because we dont know the values of q we cant determine if the secant is positive or negative and so cannot get rid of the absolute value bars here.

6. Use a trig substitution to eliminate the root in

1- 4z - 2z2 .

Hint : This doesnt look much like a term that can use a trig substitution. So, the first step should probably be to some algebraic manipulation on the quantity under the root to make it look more like a problem that can use a trig substitution. Step 1 We know that in order to do a trig substitution we really need a sum or difference of a term with a variable squared and a number. This clearly does not fit into that form. However, that doesnt mean that we cant do some algebraic manipulation on the quantity under the root to get into a form that we can do a trig substitution on. Because the quantity under the root is a quadratic polynomial we know that we can complete the square on it to turn it into something like what we need for a trig substitution. Here is the completing the square work.

1 - 4 z - 2 z 2 = -2 ( z 2 + 2 z - 1 ) 2 = -2 ( z 2 + 2 z + 1 - 1 - 1 ) 2
2 = -2 ( z + 1) - 3 2

1 ( 2 ) = [1] = 1 2
2 2

= 3 - 2 ( z + 1)

So, after completing the square the term can be written as,

1 - 4 z - 2 z 2 = 3 - 2 ( z + 1)

Hint : At this point the problem works in the same manner as the previous problems in this section. Step 2 So, in this case we see that we have a difference of a number and a squared term with a variable in it. This suggests that sine is the correct trig function to use for the substation.

2007 Paul Dawkins

34

http://tutorial.math.lamar.edu/terms.aspx

Calculus II

Now, to get the coefficient on the trig function notice that we need to turn the 2 (i.e. the coefficient of the squared term) into a 3 once weve done the substitution. With that in mind it looks like the substitution should be,

z +1 =

3 2

sin (q )

Now, all we have to do is actually perform the substitution and eliminate the root. Step 3

1 - 4 z - 2 z 2 = 3 - 2 ( z + 1) = 3 - 2
2

3 2

sin (q )

= 3 - 3sin 2 (q ) = 3 cos 2 (q ) =

3 cos (q )

Note that because we dont know the values of q we cant determine if the cosine is positive or negative and so cannot get rid of the absolute value bars here.

7. Use a trig substitution to eliminate the root in x 2 - 8 x + 21

3 2

Hint : This doesnt look much like a term that can use a trig substitution. So, the first step should probably be to some algebraic manipulation on the quantity under the root to make it look more like a problem that can use a trig substitution. Step 1 We know that in order to do a trig substitution we really need a sum or difference of a term with a variable squared and a number. This clearly does not fit into that form. However, that doesnt mean that we cant do some algebraic manipulation on the quantity under the root to get into a form that we can do a trig substitution on. Because the quantity under the root is a quadratic polynomial we know that we can complete the square on it to turn it into something like what we need for a trig substitution. Here is the completing the square work.

x 2 - 8 x + 21 = x 2 - 8 x + 16 - 16 + 21 = ( x - 4) + 5
2

1 ( -8 ) = [ -4] = 16 2
2 2

So, after completing the square the term can be written as,

(x

- 8x + 21) =
3 2

( ( x - 4 ) + 5)
2

3 2

( x - 4) + 5
2

Note that we also explicitly put the root into the problem as well.
2007 Paul Dawkins 35 http://tutorial.math.lamar.edu/terms.aspx

Calculus II

Hint : At this point the problem works in the same manner as the previous problems in this section. Step 2 So, in this case we see that we have a sum of a squared term with a variable in it and a number. This suggests that tangent is the correct trig function to use for the substation. Now, to get the coefficient on the trig function notice that we need to turn the 1 (i.e. the coefficient of the squared term) into a 5 once weve done the substitution. With that in mind it looks like the substitution should be,

x - 4 = 5 tan (q )
Now, all we have to do is actually perform the substitution and eliminate the root. Step 3
3

(x

- 8 x + 21)

3 2

3 ( x - 4) + 5 = 2 3

5 tan (q )

+ 5
3

= 5 tan 2 (q ) + 5 = 5 tan 2 (q ) + 1
3 3 = 5 sec 2 (q ) = 5 2 sec (q )

Note that because we dont know the values of q we cant determine if the secant is positive or negative and so cannot get rid of the absolute value bars here.

8. Use a trig substitution to eliminate the root in

e8 x - 9 .

Hint : This doesnt look much like a term that can use a trig substitution. So, the first step should probably be to some algebraic manipulation on the quantity under the root to make it look more like a problem that can use a trig substitution. Step 1 We know that in order to do a trig substitution we really need a sum or difference of a term with a variable squared and a number. Even though this doesnt look anything like the normal trig substitution problems it is actually pretty close to one. To see this all we need to do is rewrite the term under the root as follows.

e8 x - 9 =

(e x )
4

-9

All we did here was take advantage of the basic exponent rules to make it clear that we really do have a difference here of a squared term containing a variable and a number.

2007 Paul Dawkins

36

http://tutorial.math.lamar.edu/terms.aspx

Calculus II

Hint : At this point the problem works in the same manner as the previous problems in this section. Step 2 The form of the quantity under the root suggests that secant is the correct trig function to use for the substation. Now, to get the coefficient on the trig function notice that we need to turn the 1 (i.e. the coefficient of the squared term) into a 9 once weve done the substitution. With that in mind it looks like the substitution should be,

e 4 x = 3sec (q )
Now, all we have to do is actually perform the substitution and eliminate the root. Step 3

e8 x - 9 =

( 3sec (q ) )

- 9 = 9sec 2 (q ) - 9

= 3 sec 2 (q ) - 1 = 3 tan 2 (q ) = 3 tan (q )


Note that because we dont know the values of q we cant determine if the tangent is positive or negative and so cannot get rid of the absolute value bars here.

9. Use a trig substitution to evaluate

x 2 + 16 dx . x4

Step 1 In this case it looks like well need the following trig substitution.

x = 4 tan (q )
Now we need to use the substitution to eliminate the root and get set up for actually substituting this into the integral. Step 2 Lets first use the substitution to eliminate the root.

x 2 + 16 = 16 tan 2 (q ) + 16 = 4 sec 2 (q ) = 4 sec (q )


Next, because we are doing an indefinite integral we will assume that the secant is positive and so we can drop the absolute value bars to get,

x 2 + 16 = 4sec (q )
2007 Paul Dawkins 37 http://tutorial.math.lamar.edu/terms.aspx

Calculus II

For a final substitution preparation step lets also compute the differential so we dont forget to use that in the substitution!

dx = 4 sec 2 (q ) dq
Step 3 Now lets do the actual substitution.

4sec (q ) sec3 (q ) x 2 + 16 4sec 2 (q ) dq = dx = dq 4 16 tan 4 (q ) x4 4 tan (q ) ) (

Do not forget to substitute in the differential we computed in the previous step. This is probably the most common mistake with trig substitutions. Forgetting the differential can substantially change the problem, often making the integral very difficult to evaluate. Step 4 We now need to evaluate the integral. In this case the integral looks to be a little difficult to do in terms of secants and tangents so lets convert the integrand to sines and cosines and see what we get. Doing this gives,

x 2 + 16 1 cos (q ) dx = 4 dq x4 16 sin (q )

This is a simple integral to evaluate so here is the integral evaluation.

x 2 + 16 1 cos (q ) dx = 4 dq 4 x 16 sin (q )
1 = 16 u -4 du -3

u = sin (q )

1 1 1 = - 48 u -3 + c = - 48 sin (q ) + c = - 48 csc3 (q ) + c

Dont forget all the standard manipulations of the integrand that we often need to do in order to evaluate integrals involving trig functions. If you dont recall them youll need to go back to the previous section and work some practice problems to get good at them. Every trig substitution problem reduces down to an integral involving trig functions and the majority of them will need some manipulation of the integrand in order to evaluate. Step 5 As the final step we just need to go back to xs. To do this well need a quick right triangle. Here is that work.

2007 Paul Dawkins

38

http://tutorial.math.lamar.edu/terms.aspx

Calculus II

From the substitution we have,

tan (q ) =

x opp = 4 adj

From the right triangle we get,

csc (q ) =

x 2 + 16 x

The integral is then,

( x2 + 16 ) 2 + c x + 16 1 x 2 + 16 dx = - +c = 48 48 x 3 x4 x
2 3
3

10. Use a trig substitution to evaluate

1 - 7w2 dw .

Step 1 In this case it looks like well need the following trig substitution.

w=

1 7

sin (q )

Now we need to use the substitution to eliminate the root and get set up for actually substituting this into the integral. Step 2 Lets first use the substitution to eliminate the root.

1 - 7 w2 = 1 - sin 2 (q ) = cos 2 (q ) = cos (q )


Next, because we are doing an indefinite integral we will assume that the cosine is positive and so we can drop the absolute value bars to get,

1 - 7 w2 = cos (q )
For a final substitution preparation step lets also compute the differential so we dont forget to use that in the substitution!

dw =
Step 3 Now lets do the actual substitution.
2007 Paul Dawkins

1 7

cos (q ) dq

39

http://tutorial.math.lamar.edu/terms.aspx

Calculus II

1 - 7 w2 dw = cos (q )

1 7

cos (q ) dq =

1 7

cos (q ) dq
2

Do not forget to substitute in the differential we computed in the previous step. This is probably the most common mistake with trig substitutions. Forgetting the differential can substantially change the problem, often making the integral very difficult to evaluate. Step 4 We now need to evaluate the integral. Here is that work.

1 - 7 w2 dw =

1 7

1 2

1 + cos ( 2q ) dq =

1 2 7

q + 1 sin ( 2q ) + c 2

Dont forget all the standard manipulations of the integrand that we often need to do in order to evaluate integrals involving trig functions. If you dont recall them youll need to go back to the previous section and work some practice problems to get good at them. Every trig substitution problem reduces down to an integral involving trig functions and the majority of them will need some manipulation of the integrand in order to evaluate. Step 5 As the final step we just need to go back to ws. To eliminate the the first term (i.e. the q ) we can use any of the inverse trig functions. The easiest is to probably just use the original substitution and get a formula involving inverse sine but any of the six trig functions could be used if we wanted to. Using the substitution gives us,

sin (q ) = 7 w

q = sin -1

7w

Eliminating the sin ( 2q ) requires a little more work. We cant just use a right triangle as we normally would because that would only give trig functions with an argument of q and we have an argument of 2q . However, we could use the double angle formula for sine to reduce this to trig functions with arguments of q . Doing this gives,

1 - 7 w2 dw =

1 2 7

q + sin (q ) cos (q ) + c

We can now do the right triangle work.

2007 Paul Dawkins

40

http://tutorial.math.lamar.edu/terms.aspx

Calculus II

From the substitution we have,

sin (q ) =

7 w opp = 1 hyp

From the right triangle we get,

cos (q ) = 1 - 7w2

The integral is then,

1 - 7 w2 dw =

1 2 7

sin -1

7 w + 7 w 1 - 7 w2 + c

11. Use a trig substitution to evaluate

2 3 2 t (3t - 4 ) dt . 5

Step 1 First, do not get excited about the exponent in the integrand. These types of problems work exactly the same as those with just a root (as opposed to this case in which we have a root to a power you do agree that is what we have right?). So, in this case it looks like well need the following trig substitution.

t=

2 3

sec (q )

Now we need to use the substitution to eliminate the root and get set up for actually substituting this into the integral. Step 2 Lets first use the substitution to eliminate the root.

( 3t

5 - 4 ) 2 = 3t 2 - 4 = 4sec 2 (q ) - 4 = 2 tan 2 (q ) = 32 tan (q )


5

Next, because we are doing an indefinite integral we will assume that the tangent is positive and so we can drop the absolute value bars to get,

( 3t

- 4 ) 2 = 32 tan 5 (q )
5

For a final substitution preparation step lets also compute the differential so we dont forget to use that in the substitution!

dt =

2 3

sec (q ) tan (q ) dq
41 http://tutorial.math.lamar.edu/terms.aspx

2007 Paul Dawkins

Calculus II

Step 3 Now lets do the actual substitution.

t ( 3t
3

- 4 ) 2 dt =
5

( ) sec (q ) (32 tan


2 3 3 3

(q ) ) ( 23 sec (q ) tan (q ) ) dq

= 512 sec 4 (q ) tan 6 (q ) dq 9


Do not forget to substitute in the differential we computed in the previous step. This is probably the most common mistake with trig substitutions. Forgetting the differential can substantially change the problem, often making the integral very difficult to evaluate. Step 4 We now need to evaluate the integral. Here is that work.

t ( 3t
3

- 4 ) 2 dt = 512 ( tan 2 (q ) + 1) tan 6 (q ) sec 2 (q ) dq 9


5

u = tan (q )

= 512 ( u 2 + 1) u 6 du = 9

512 9

+ u 6 du

= 512 1 tan 9 (q ) + 1 tan 7 (q ) + c 9 9 7


Dont forget all the standard manipulations of the integrand that we often need to do in order to evaluate integrals involving trig functions. If you dont recall them youll need to go back to the previous section and work some practice problems to get good at them. Every trig substitution problem reduces down to an integral involving trig functions and the majority of them will need some manipulation of the integrand in order to evaluate. Step 5 As the final step we just need to go back to ts. To do this well need a quick right triangle. Here is that work. From the substitution we have,

sec (q ) =

3 t hyp = 2 adj

From the right triangle we get,

3t 2 - 4 tan (q ) = 2
The integral is then,

2007 Paul Dawkins

42

http://tutorial.math.lamar.edu/terms.aspx

Calculus II

9 7 512 1 3t 2 - 4 1 3t 2 - 4 t ( 3t - 4 ) dt = + +c 7 9 9 2 2 3 2
5 2

( 3t 2 - 4 ) 2
9

81

4 ( 3t 2 - 4 ) 2
7

63

+c

12. Use a trig substitution to evaluate

-5 4

2 y 2 - 25

-7 y

dy .

Step 1 In this case it looks like well need the following trig substitution.

y = 5sec (q )
Now we need to use the substitution to eliminate the root and get set up for actually substituting this into the integral. Step 2 Lets first use the substitution to eliminate the root.

y 2 - 25 = 25sec 2 (q ) - 25 = 5 tan 2 (q ) = 5 tan (q )


Step 3 Okay, in this case we have limits on y and so we can get limits on q that will allow us to determine if tangent is positive or negative to allow us to eliminate the absolute value bars. So, lets get some limits on q .

y = -7 : - 7 = 5sec (q ) y = -5 : - 5 = 5sec (q )

sec (q ) = - 7 5 sec (q ) = -1

q = sec -1 ( - 7 ) = 2.3664 5 q =p

So, q s for this problem are in the range 2.3664 q p and these are in the second quadrant. In the second quadrant we know that tangent is negative and so we can drop the absolute value bars provided we add in a minus sign. This gives,

y 2 - 25 = -5 tan (q )
For a final substitution preparation step lets also compute the differential so we dont forget to use that in the substitution!

2007 Paul Dawkins

43

http://tutorial.math.lamar.edu/terms.aspx

Calculus II

dy = 5sec (q ) tan (q ) dq
Step 4 Now lets do the actual substitution.

2 2 dy = ( 5sec (q ) tan (q ) ) dq 4 4 2.3664 5 sec (q ) ( -5 tan (q ) ) -7 y 4 y 2 - 25 2 1 =dq 3 625 2.3664 sec (q )


Do not forget to substitute in the differential we computed in the previous step. This is probably the most common mistake with trig substitutions. Forgetting the differential can substantially change the problem, often making the integral very difficult to evaluate. Also notice that upon doing the substation we replaced the y limits with the q limits. This will help with a later step. Step 5 We now need to evaluate the integral. In terms of secants this integral would be pretty difficult, however we a quick change to cosines we get the following integral.
p

-5

2 2 p dy = cos3 (q ) dq 4 2 625 2.3664 -7 y y - 25


This should be relatively simple to do so here is the integration work.

-5

2 2 p dy = (1 - sin 2 (q ) ) cos (q ) dq 4 2 625 2.3664 -7 y y - 25


2 = - 625 sin (p ) sin ( 2.3664 )

-5

u = sin (q )

1 - u 2 du
0 0.69986

2 = - 625 u - 1 u 3 3

= 0.001874

Dont forget all the standard manipulations of the integrand that we often need to do in order to evaluate integrals involving trig functions. If you dont recall them youll need to go back to the previous section and work some practice problems to get good at them. Every trig substitution problem reduces down to an integral involving trig functions and the majority of them will need some manipulation of the integrand in order to evaluate. Also, note that because we converted the limits at every substitution into limits for the new variable we did not need to do any back substitution work on our answer!

2007 Paul Dawkins

44

http://tutorial.math.lamar.edu/terms.aspx

Calculus II

13. Use a trig substitution to evaluate

2 z 5 2 + 9 z 2 dz .

Step 1 In this case it looks like well need the following trig substitution.

z=

2 3

tan (q )

Now we need to use the substitution to eliminate the root and get set up for actually substituting this into the integral. Step 2 Lets first use the substitution to eliminate the root.

2 + 9 z 2 = 2 + 2 tan 2 (q ) = 2 sec 2 (q ) = 2 sec (q )


Step 3 Okay, in this case we have limits on z and so we can get limits on q that will allow us to determine if tangent is positive or negative to allow us to eliminate the absolute value bars. So, lets get some limits on q .

z = 1: 1 = z = 4: 4 =

2 3 2 3

tan (q ) tan (q )

tan (q ) = tan (q ) =

3 2 12 2

q = tan -1

q = tan -1

( ) = 1.1303 ( ) = 1.4535
3 2 12 2

So, q s for this problem are in the range 1.1303 q 1.4535 and these are in the first quadrant. In the first quadrant we know that cosine, and hence secant, is positive and so we can just drop the absolute value bars. This gives,

2 + 9 z 2 = 2 sec (q )
For a final substitution preparation step lets also compute the differential so we dont forget to use that in the substitution!

dz =
Step 4 Now lets do the actual substitution.

2 3

sec 2 (q ) dq

2 z 5 2 + 9 z 2 dz = 2 1.1303 = 16 2 729

1.4535

( )
2 3

tan 5 (q )

2 sec (q )

)(

2 3

sec 2 (q ) dq

1.4535

1.1303

tan 5 (q ) sec3 (q ) dq

2007 Paul Dawkins

45

http://tutorial.math.lamar.edu/terms.aspx

Calculus II

Do not forget to substitute in the differential we computed in the previous step. This is probably the most common mistake with trig substitutions. Forgetting the differential can substantially change the problem, often making the integral very difficult to evaluate. Also notice that upon doing the substation we replaced the y limits with the q limits. This will help with a later step. Step 5 We now need to evaluate the integral. Here is that work.

2 z 5 2 + 9 z 2 dz = 16 2 729 = 16 2 729 = 16 2 729

sec 2 (q ) - 1 sec 2 (q ) tan (q ) sec (q ) dq 1.1303


2 sec(1.4535 ) sec(1.1303) 8.5444

1.4535

u 2 - 1 u 2 du

u = sec (q )

2.3452

u 6 - 2u 4 + u 2 du
8.5444 2.3452

1 2 = 16 2 7 u 7 - 5 u 5 + 1 u 3 729 3

= 14182.86074

Dont forget all the standard manipulations of the integrand that we often need to do in order to evaluate integrals involving trig functions. If you dont recall them youll need to go back to the previous section and work some practice problems to get good at them. Every trig substitution problem reduces down to an integral involving trig functions and the majority of them will need some manipulation of the integrand in order to evaluate. Also, note that because we converted the limits at every substitution into limits for the new variable we did not need to do any back substitution work on our answer!

14. Use a trig substitution to evaluate

1 dx . 2 9 x - 36 x + 37

Step 1 The first thing well need to do here is complete the square on the polynomial to get this into a form we can use a trig substitution on.
2 1 9 x 2 - 36 x + 37 = 9 ( x 2 - 4 x + 37 ) = 9 ( x 2 - 4 x + 4 - 4 + 37 ) = 9 ( x - 2 ) + 9 9 9

= 9 ( x - 2) + 1
2

The integral is now,

1 1 dx = dx 2 2 9 x - 36 x + 37 9 ( x - 2) + 1
2007 Paul Dawkins 46 http://tutorial.math.lamar.edu/terms.aspx

Calculus II

Now we can proceed with the trig substitution. Step 2 It looks like well need to the following trig substitution.

x - 2 = 1 tan (q ) 3
Next lets eliminate the root.

9 ( x - 2 ) + 1 = tan (q ) + 1 = sec2 (q ) = sec (q )


2 2

Next, because we are doing an indefinite integral we will assume that the secant is positive and so we can drop the absolute value bars to get,

9 ( x - 2 ) + 1 = sec (q )
2

For a final substitution preparation step lets also compute the differential so we dont forget to use that in the substitution!

(1) dx = 1 sec2 (q ) dq 3

dx = 1 sec 2 (q ) dq 3

Recall that all we really need to do here is compute the differential for both the right and left sides of the substitution. Step 3 Now lets do the actual substitution.

1 1 1 1 2 dx = sec (q ) dq = sec (q ) dq 2 3 9 x - 36 x + 37 sec (q ) 3


Do not forget to substitute in the differential we computed in the previous step. This is probably the most common mistake with trig substitutions. Forgetting the differential can substantially change the problem, often making the integral very difficult to evaluate. Step 4 We now need to evaluate the integral. Here is that work.

1 1 dx = ln sec (q ) + tan (q ) + c 2 3 9 x - 36 x + 37
Note that this was one of the few trig substitution integrals that didnt really require a lot of manipulation of trig functions to completely evaluate. All we had to really do here was use the fact that we determined the integral of sec (q ) in the previous section and reuse that result here. Step 5
2007 Paul Dawkins 47 http://tutorial.math.lamar.edu/terms.aspx

Calculus II

As the final step we just need to go back to xs. To do this well need a quick right triangle. Here is that work. From the substitution we have,

tan (q ) =

3 ( x - 2 ) opp = 1 adj

From the right triangle we get,

sec (q ) = 9 ( x - 2 ) + 1
2

The integral is then,

1 1 2 dx = ln 9 ( x - 2 ) + 1 + 3 ( x - 2 ) + c 2 3 9 x - 36 x + 37

15. Use a trig substitution to evaluate

( z + 3)

40 - 6 z - z 2 3 )2 (

dz .

Step 1 The first thing well need to do here is complete the square on the polynomial to get this into a form we can use a trig substitution on.
2 40 - 6 z - z 2 = - ( z 2 + 6 z - 40 ) = - ( z 2 + 6 z + 9 - 9 - 40 ) = - ( z + 3) - 49

= 49 - ( z + 3)
The integral is now,

5 5 ( z + 3) ( z + 3) dz = 2 40 - 6 z - z 2 3 49 - z + 3 2 ) ( ( )

3 2

dz

Now we can proceed with the trig substitution. Step 2 It looks like well need to the following trig substitution.

z + 3 = 7 sin (q )
Next lets eliminate the root.
2007 Paul Dawkins 48 http://tutorial.math.lamar.edu/terms.aspx

Calculus II

( 49 - ( z + 3) )

2 2

3 = 49 - ( z + 3) = 49 - 49sin 2 (q ) = 7 cos 2 (q ) = 343 cos (q )

Next, because we are doing an indefinite integral we will assume that the cosine is positive and so we can drop the absolute value bars to get,

49 - ( z + 3)

2 2

= 343cos3 (q )

For a final substitution preparation step lets also compute the differential so we dont forget to use that in the substitution!

(1) dz = 7 cos (q ) dq

dz = 7 cos (q ) dq

Recall that all we really need to do here is compute the differential for both the right and left sides of the substitution. Step 3 Now lets do the actual substitution.
5 16807 sin 5 (q ) sin 5 (q ) ( z + 3) dz = ( 7 cos (q ) ) dq = 343 cos2 q dq 3 2 40 - 6 z - z 2 3 ( ) 343cos (q ) ) (

Do not forget to substitute in the differential we computed in the previous step. This is probably the most common mistake with trig substitutions. Forgetting the differential can substantially change the problem, often making the integral very difficult to evaluate. Step 4 We now need to evaluate the integral. Here is that work.
5 1 - cos 2 (q ) ( z + 3) sin q dq dz = 343 ( ) 2 2 40 - 6 z - z 2 3 cos (q ) ) ( 2 1 - u du = -343 u -2 - 2 + u 2 du = -343 2 u 2 2

u = cos (q )

= -343 ( -u -1 - 2u + 1 u 3 ) + c 3

1 1 = -343 - 2 cos (q ) + cos3 (q ) + c cos (q ) 3


Dont forget all the standard manipulations of the integrand that we often need to do in order to evaluate integrals involving trig functions. If you dont recall them youll need to go back to the previous section and work some practice problems to get good at them.
2007 Paul Dawkins 49 http://tutorial.math.lamar.edu/terms.aspx

Calculus II

Every trig substitution problem reduces down to an integral involving trig functions and the majority of them will need some manipulation of the integrand in order to evaluate. Step 5 As the final step we just need to go back to zs. To do this well need a quick right triangle. Here is that work. From the substitution we have,

sin (q ) =

z + 3 adj = 7 hyp 49 - ( z + 3) 7
2

From the right triangle we get,

cos (q ) =

The integral is then,


5 ( z + 3) dz = 2 40 - 6 z - z 2 3 ) (

2401 49 - ( z + 3)
2

+ 98 49 - ( z + 3 ) 2

49 - ( z + 3) 3

2 2

+c

16. Use a trig substitution to evaluate

cos ( x )

9 + 25sin 2 ( x ) dx .

Step 1 Lets first rewrite the integral a little bit.

cos ( x )

9 + 25sin 2 ( x ) dx = cos ( x ) 9 + 25 sin ( x ) dx


2

Step 2 With the integral written as it is in the first step we can now see that we do have a sum of a number and something squared under the root. We know from the problems done previously in this section that looks like a tangent substitution. So, lets use the following substitution.
3 sin ( x ) = 5 tan (q )

Do not get excited about the fact that we are substituting one trig function for another. That will happen on occasion with these kinds of problems. Note however, that we need to be careful and make sure that we also change the variable from x (i.e. the variable in the original trig function) into q (i.e. the variable in the new trig function).

2007 Paul Dawkins

50

http://tutorial.math.lamar.edu/terms.aspx

Calculus II

Next lets eliminate the root.

9 + 25 sin ( x ) = 9 + 25 3 tan (q ) = 9 + 9 tan 2 (q ) = 3 sec2 (q ) = 3 sec (q ) 5


2 2

Next, because we are doing an indefinite integral we will assume that the secant is positive and so we can drop the absolute value bars to get,

9 + 25 sin ( x ) = 3sec (q )
2

For a final substitution preparation step lets also compute the differential so we dont forget to use that in the substitution!

cos ( x ) dx = 3 sec 2 (q ) dq 5
Recall that all we really need to do here is compute the differential for both the right and left sides of the substitution. Step 3 Now lets do the actual substitution.
2 cos ( x ) 9 + 25sin ( x ) dx = 9 + 25 sin ( x ) 2

cos ( x ) dx

= ( 3sec (q ) ) ( 3 sec2 (q ) ) dq = 9 sec3 (q ) dq 5 5

Do not forget to substitute in the differential we computed in the previous step. This is probably the most common mistake with trig substitutions. Forgetting the differential can substantially change the problem, often making the integral very difficult to evaluate. Step 4 We now need to evaluate the integral. Here is that work.

cos ( x )

9 9 + 25sin 2 ( x ) dx = 10 sec (q ) tan (q ) + ln sec (q ) + tan (q ) + c

Note that this was one of the few trig substitution integrals that didnt really require a lot of manipulation of trig functions to completely evaluate. All we had to really do here was use the 3 fact that we determined the integral of sec (q ) in the previous section and reuse that result here. Step 5 As the final step we just need to go back to zs. To do this well need a quick right triangle. Here is that work.

2007 Paul Dawkins

51

http://tutorial.math.lamar.edu/terms.aspx

Calculus II

From the substitution we have,

tan (q ) =

5sin ( x ) opp = 3 adj


9 + 25sin 2 ( x ) 3

From the right triangle we get,

sec (q ) =
The integral is then,

cos ( x )

9 + 25sin

( x ) dx =

sin ( x ) 9 + 25sin 2 ( x ) 2

5sin ( x ) + 9 + 25sin 2 ( x ) 9 + ln +c 10 3

Partial Fractions
1. Evaluate the integral

4 dx . x + 5 x - 14
2

Step 1 To get the problem started off we need the form of the partial fraction decomposition of the integrand. However, in order to get this well need to factor the denominator.

4 4 dx = dx 2 x + 5 x - 14 ( x + 7 )( x - 2 )
The form of the partial fraction decomposition for the integrand is then,

( x + 7 )( x - 2 )
Step 2 Setting the numerators equal gives,

A B + x+7 x-2

4 = A ( x - 2) + B ( x + 7 )
Step 3
2007 Paul Dawkins 52 http://tutorial.math.lamar.edu/terms.aspx

Calculus II

We can use the trick discussed in the notes to easily get the coefficients in this case so lets do that. Here is that work.

x = 2 : 4 = 9B x = -7 : 4 = -9 A
The partial fraction form of the integrand is then,

4 A=-9 4 B=9

( x + 7 )( x - 2 )
Step 4 We can now do the integral.

4 4 -9 + 9 x+7 x-2

4 -4 4 4 4 dx = 9 + 9 dx = ln x - 2 - ln x + 7 + c 9 9 x+7 x-2 ( x + 7 )( x - 2 )

2. Evaluate the integral

8 - 3t dt . 10t + 13t - 3
2

Step 1 To get the problem started off we need the form of the partial fraction decomposition of the integrand. However, in order to get this well need to factor the denominator.

8 - 3t 8 - 3t dt = dt 2 10t + 13t - 3 ( 2t + 3 )( 5t - 1)
The form of the partial fraction decomposition for the integrand is then,

8 - 3t A B = + 10t + 13t - 3 2t + 3 5t - 1
2

Step 2 Setting the numerators equal gives,

8 - 3t = A ( 5t - 1) + B ( 2t + 3)
Step 3 We can use the trick discussed in the notes to easily get the coefficients in this case so lets do that. Here is that work.

t= 1: 5 3 t=-2:
2007 Paul Dawkins

37 5 25 2

= 17 B 5 = - 17 A 2
53

25 A = - 17 37 B = 17

http://tutorial.math.lamar.edu/terms.aspx

Calculus II

The partial fraction form of the integrand is then,


37 - 25 8 - 3t = 17 + 17 10t 2 + 13t - 3 2t + 3 5t - 1

Step 4 We can now do the integral.


37 - 25 8 - 3t 37 25 dt = 17 + 17 dt = ln 5t - 1 - ln 2t + 3 + c 2 10t + 13t - 3 85 34 2t + 3 5t - 1

Hopefully you are getting good enough with integration that you can do some of these integrals in your head. Be careful however with both of these integrals. When doing these kinds of integrals in our head it is easy to forget about the substitutions that are technically required to do them and then miss the coefficients from the substitutions that need to show up in the answer.

w2 + 7 w 3. Evaluate the integral dw . -1 ( w + 2 )( w - 1)( w - 4 )


Step 1 In this case the denominator is already factored and so we can go straight to the form of the partial fraction decomposition for the integrand.

w2 + 7 w A B C = + + ( w + 2 )( w - 1)( w - 4 ) w + 2 w - 1 w - 4
Step 2 Setting the numerators equal gives,

w2 + 7 w = A ( w - 1)( w - 4 ) + B ( w + 2 )( w - 4 ) + C ( w + 2 )( w - 1)
Step 3 We can use the trick discussed in the notes to easily get the coefficients in this case so lets do that. Here is that work.

w = 1: 8 = -9 B w = 4: 44 = 18C w = -2 : -10 = 18 A
The partial fraction form of the integrand is then,

5 A=-9 B=-8 9 22 C= 9

2007 Paul Dawkins

54

http://tutorial.math.lamar.edu/terms.aspx

Calculus II

5 8 22 -9 w2 + 7 w = - 9 + 9 ( w + 2 )( w - 1)( w - 4 ) w + 2 w - 1 w - 4

Step 4 We can now do the integral.


0 5 8 22 w2 + 7 w - 9 - 9 + 9 dw dw = w + 2 w -1 w - 4 -1 w + 2 w - 1 w - 4 )( )( ) -1 ( 0

= ( - 5 ln w + 2 - 8 ln w - 1 + 22 ln w - 4 ) 9 9 9 =
22 9 3 ln ( 4 ) + 9 ln ( 2 ) - 22 ln ( 5 ) = 9 47 9

0 -1

ln ( 2 ) - 22 ln ( 5 ) 9

Note that we used a quick logarithm property to combine the first two logarithms into a single logarithm. You should probably review your logarithm properties if you dont recognize the one that we used. These kinds of property applications can really simplify your work on occasion if you know them!

4. Evaluate the integral

8 dx . 3x + 7 x 2 + 4 x
3

Step 1 To get the problem started off we need the form of the partial fraction decomposition of the integrand. However, in order to get this well need to factor the denominator.

8 8 dx = dx 3 2 3x + 7 x + 4 x x ( 3 x + 4 )( x + 1)
The form of the partial fraction decomposition for the integrand is then,

8 A B C = + + x ( 3x + 4 )( x + 1) x 3x + 4 x + 1
Step 2 Setting the numerators equal gives,

8 = A ( 3x + 4 )( x + 1) + Bx ( x + 1) + C x ( 3x + 4 )
Step 3 We can use the trick discussed in the notes to easily get the coefficients in this case so lets do that. Here is that work.

2007 Paul Dawkins

55

http://tutorial.math.lamar.edu/terms.aspx

Calculus II
4 x=-4: 8= 9B 3 x = -1: 8 = -C

A=2 B = 18 C = -8

x = 0:

8 = 4A

The partial fraction form of the integrand is then,

8 2 18 8 = + x ( 3x + 4 )( x + 1) x 3 x + 4 x + 1
Step 4 We can now do the integral.

8 2 18 8 dx = + dx = 2ln x + 6 ln 3x + 4 - 8ln x + 1 + c x 3x + 4 x + 1 x ( 3x + 4 )( x + 1)
Hopefully you are getting good enough with integration that you can do some of these integrals in your head. Be careful however with the second integral. When doing these kinds of integrals in our head it is easy to forget about the substitutions that are technically required to do them and then miss the coefficients from the substitutions that need to show up in the answer.

3z 2 + 1 5. Evaluate the integral dz . 2 2 ( z + 1)( z - 5 )


Step 1 In this case the denominator is already factored and so we can go straight to the form of the partial fraction decomposition for the integrand.

3z 2 + 1

( z + 1)( z - 5)
Step 2 Setting the numerators equal gives,

A B C + + z + 1 z - 5 ( z - 5 )2

3z 2 + 1 = A ( z - 5) + B ( z + 1)( z - 5) + C ( z + 1)
2

Step 3 We can use the trick discussed in the notes to easily get two of the coefficients and then we can just pick another value of z to get the third so lets do that. Here is that work.

2007 Paul Dawkins

56

http://tutorial.math.lamar.edu/terms.aspx

Calculus II

z = -1: 4 = 36 A z = 5 : 76 = 6C z = 0: 1 = 25 A - 5B + C = 139 - 5B 9
The partial fraction form of the integrand is then,

A= B= C=

1 9 26 9 38 3

3z 2 + 1

( z + 1)( z - 5)
Step 4 We can now do the integral.

1 9

z +1 z - 5

26 9

( z - 5)

38 3

26 38 1 3z 2 + 1 3 dz = 9 + 9 + dz 2 2 2 z + 1 z - 5 ( z - 5) 2 ( z + 1)( z - 5) 38 = 1 ln z + 1 + 26 ln z - 5 - 3 = 1 ln ( 5) - 27 ln ( 3) + 76 9 9 9 9 9 z -5 2 4

4 x - 11 6. Evaluate the integral 3 dx . 2 x - 9x


Step 1 To get the problem started off we need the form of the partial fraction decomposition of the integrand. However, in order to get this well need to factor the denominator.

4 x - 11 dx = 4 x - 11 dx 3 2 x - 9x2 x ( x - 9)
The form of the partial fraction decomposition for the integrand is then,

4 x - 11 A B C = + 2+ 2 x ( x - 9) x x x-9
Step 2 Setting the numerators equal gives,

4 x - 11 = Ax ( x - 9 ) + B ( x - 9 ) + C x 2
Step 3 We can use the trick discussed in the notes to easily get two of the coefficients and then we can just pick another value of x to get the third so lets do that. Here is that work.
2007 Paul Dawkins 57 http://tutorial.math.lamar.edu/terms.aspx

Calculus II

x = 0 : -11 = -9 B x = 9 : 25 = 81C x = 1: -7 = -8 A - 8 B + C = -8 A - 767 81


The partial fraction form of the integrand is then,
25 - 25 11 4 x - 11 = 81 + 92 + 81 2 x ( x - 9) x x x-9

A = - 25 81 11 B= 9 C = 25 81

Step 4 We can now do the integral.


25 11 - 25 11 4 x - 11 dx = 81 + 92 + 81 dx = - 25 ln x - 9 + 25 ln x - 9 + c 2 81 x x -9 x 81 x x ( x - 9)

7. Evaluate the integral

z2 + 2z + 3 dz . ( z - 6) ( z 2 + 4)

Step 1 In this case the denominator is already factored and so we can go straight to the form of the partial fraction decomposition for the integrand.

z2 + 2z + 3 A Bz + C = + 2 2 ( z - 6) ( z + 4) z - 6 z + 4
Step 2 Setting the numerators equal gives,

z 2 + 2 z + 3 = A ( z 2 + 4 ) + ( Bz + C )( z - 6 ) = ( A + B ) z 2 + ( -6 B + C ) z + 4 A - 6C
In this case the trick discussed in the notes wont work all that well for us and so well have to resort to multiplying everything out and collecting like terms as shown above. Step 3 Now, setting the coefficients equal gives the following system.

z2 :

A+ B =1

A= C=

51 40

z1 : -6 B + C = 2 z 0 : 4 A - 6C = 3
The partial fraction form of the integrand is then,
2007 Paul Dawkins 58

B = - 11 40
7 20

http://tutorial.math.lamar.edu/terms.aspx

Calculus II

51 7 - 11 z + 20 z2 + 2z + 3 = 40 + 402 ( z - 6) ( z 2 + 4) z - 6 z + 4

Step 4 We can now do the integral.


51 7 z2 + 2z + 3 - 11 z + 20 dz = 40 + 402 dz 2 z +4 z-6 ( z - 6) ( z + 4) 51 7 - 11 z = 40 + 2 40 + 2 20 dz z-6 z +4 z +4

51 40

7 z 11 ln z - 6 - 80 ln z 2 + 4 + 40 tan -1 ( 2 ) + c

Note that the second integration needed the substitution u = z 2 + 4 while the third needed the formula provided in the notes.

8 + t + 6t 2 - 12t 3 8. Evaluate the integral dt . 2 2 ( 3t + 4 )( t + 7 )


Step 1 In this case the denominator is already factored and so we can go straight to the form of the partial fraction decomposition for the integrand.

8 + t + 6t 2 - 12t 3 At + B Ct + D = + ( 3t 2 + 4 )( t 2 + 7 ) 3t 2 + 4 t 2 + 7
Step 2 Setting the numerators equal gives,

8 + t + 6t 2 - 12t 3 = ( At + B ) ( t 2 + 7 ) + ( Ct + D ) ( 3t 2 + 4 ) = ( A + 3C ) t 3 + ( B + 3D ) t 2 + ( 7 A + 4C ) t + 7 B + 4 D
In this case the trick discussed in the notes wont work all that well for us and so well have to resort to multiplying everything out and collecting like terms as shown above. Step 3 Now, setting the coefficients equal gives the following system.

2007 Paul Dawkins

59

http://tutorial.math.lamar.edu/terms.aspx

Calculus II

t3 : A + 3C = -12 2 t : B + 3D = 6 t1 : 7 A + 4C = 1 t 0 : 7B + 4D = 8
The partial fraction form of the integrand is then,

A=3 B=0 C = -5 D=2

8 + t + 6t 2 - 12t 3 3t -5t + 2 = 2 + 2 2 2 ( 3t + 4 )( t + 7 ) 3t + 4 t + 7
Step 4 We can now do the integral.

8 + t + 6t 2 - 12t 3 -5t + 2 3t + 2 dt = 2 dt 2 2 3t + 4 t + 7 ( 3t + 4 )( t + 7 ) 3t 5t 2 = 2 - 2 + 2 dt 3t + 4 t + 7 t + 7 =
1 2

ln 3t 2 + 4 - 5 ln t 2 + 7 + 2

2 7

tan -1

( )+c
t
7

Note that the first and second integrations needed the substitutions u = 3t 2 + 4 and u = t 2 + 7 respectively while the third needed the formula provided in the notes.

9. Evaluate the integral dx . ( x - 2 )( x + 4 ) Hint : Pay attention to the degree of the numerator and denominator! Step 1 Remember that we can only do partial fractions on a rational expression if the degree of the numerator is less than the degree of the denominator. In this case both the numerator and denominator are both degree 2. This can be easily seen if we multiply the denominator out.

6 x 2 - 3x

6 x 2 - 3x 6 x 2 - 3x = ( x - 2 )( x + 4 ) x 2 + 2 x - 8
So, the first step is to do long division (well leave it up to you to check our Algebra skills for the long division) to get,

2007 Paul Dawkins

60

http://tutorial.math.lamar.edu/terms.aspx

Calculus II

6 x 2 - 3x 48 - 15 x = 6+ ( x - 2 )( x + 4 ) ( x - 2 )( x + 4 )
Step 2 Now we can do the partial fractions on the second term. Here is the form of the partial fraction decomposition.

48 - 15 x A B = + ( x - 2 )( x + 4 ) x - 2 x + 4
Setting the numerators equal gives,

48 - 15 x = A ( x + 4 ) + B ( x - 2 )
Step 3 The trick will work here easily enough so here is that work.

x = -4 : 108 = -6 B x = 2: 18 = 6 A

A=3 B = -18

The partial fraction form of the second term is then,

48 - 15 x 3 18 = ( x - 2 )( x + 4 ) x - 2 x + 4
Step 4 We can now do the integral.

6 x 2 - 3x 3 18 x - 2 x + 4 dx = 6 + x - 2 - x + 4 dx = 6 x + 3ln x - 2 - 18ln x + 4 + c )( ) (

4 2 + w dw . 10. Evaluate the integral 3 w + 9w

Hint : Pay attention to the degree of the numerator and denominator! Step 1 Remember that we can only do partial fractions on a rational expression if the degree of the numerator is less than the degree of the denominator. In this case the degree of the numerator is 4 and the degree of the denominator is 3. So, the first step is to do long division (well leave it up to you to check our Algebra skills for the long division) to get,
2007 Paul Dawkins 61 http://tutorial.math.lamar.edu/terms.aspx

Calculus II

2 + w4 2 - 9 w2 = w+ w3 + 9 w w ( w2 + 9 )
Step 2 Now we can do the partial fractions on the second term. Here is the form of the partial fraction decomposition.

2 - 9 w2 A Bw + C = + 2 2 w ( w + 9) w w + 9
Setting the numerators equal gives,

2 - 9w2 = A ( w2 + 9 ) + w ( Bw + C ) = ( A + B ) w2 + Cw + 9 A
In this case the trick discussed in the notes wont work all that well for us and so well have to resort to multiplying everything out and collecting like terms as shown above. Step 3 Now, setting the coefficients equal gives the following system.

w 2 : A + B = -9 w1 : C=0 0 w : 9A = 2
The partial fraction form of the second term is then,

2 A= 9 B = - 83 9 C =0

83 2 w 2 - 9 w2 = 9 - 9 2 2 w ( w + 9) w w + 9

Step 4 We can now do the integral.


4 2 2 + w dw = w + 9 - 83 w dw = 9 3 w w2 + 9 w + 9w 1 2 83 2 w2 + 9 ln w - 18 ln w2 + 9 + c

Integrals Involving Roots

2007 Paul Dawkins

62

http://tutorial.math.lamar.edu/terms.aspx

Calculus II

1. Evaluate the integral

7 dx . 2+ x-4

Step 1 The substitution well use here is,

u = x-4
Step 2 Now we need to get set up for the substitution. In other words, we need so solve for x and get dx.

x = u2 + 4
Step 3 Doing the substitution gives,

dx = 2u du

7 7 14u dx = du ( 2u ) du = 2+u 2+u 2+ x-4


Step 4 This new integral can be done with the substitution v = u + 2 . Doing this gives,

14 ( v - 2 ) 7 28 dx = dv = 14 - dv = 14v - 28ln v + c v v 2+ x-4


Step 5 The last step is to now do all the back substitutions to get the final answer.

7 dx = 14 ( u + 2 ) - 28ln u + 2 + c = 14 2+ x-4

x - 4 + 2 - 28ln

x-4 +2 +c

Note that we could have avoided the second substitution if wed used u = original substitution.

x - 4 + 2 for the

This often doesnt work, but in this case because the only extra term in the denominator was a constant it didnt change the differential work and so would work pretty easily for this problem.

2. Evaluate the integral

1 dw . w + 2 1- w + 2

Step 1 The substitution well use here is,

2007 Paul Dawkins

63

http://tutorial.math.lamar.edu/terms.aspx

Calculus II

u = 1- w
Step 2 Now we need to get set up for the substitution. In other words, we need so solve for w and get dw.

w = 1- u2
Step 3 Doing the substitution gives,

dw = -2u du

1 1 2u dw = du ( -2u ) du = 2 2 1 - u + 2u + 2 u - 2u - 3 w + 2 1- w + 2
Step 4 This integral requires partial fractions to evaluate. Lets start with the form of the partial fraction decomposition.

2u A B = + ( u + 1)( u - 3) u + 1 u - 3
Setting the coefficients equal gives,

2u = A ( u - 3) + B ( u + 1)
Using the trick to get the coefficients gives,

u = 3: 6 = 4B u = -1: -2 = -4 A
The integral is then,

A= B=

1 2 3 2

1 3 2u 1 3 du = 2 + 2 du = ln u + 1 + ln u - 3 + c u +1 u - 3 2 2 ( u + 1)( u - 3)

Step 5 The last step is to now do all the back substitutions to get the final answer.

1 1 3 dw = ln 1 - w + 1 + ln 1 - w - 3 + c 2 2 w + 2 1- w + 2

2007 Paul Dawkins

64

http://tutorial.math.lamar.edu/terms.aspx

Calculus II

3. Evaluate the integral

t -2 dt . t - 3 2t - 4 + 2

Step 1 The substitution well use here is,

u = 2t - 4
Step 2 Now we need to get set up for the substitution. In other words, we need so solve for t and get dt.

t = 1 u2 + 2 2
Step 3 Doing the substitution gives,

dt = u du

t -2 u3 1 u2 + 2 - 2 dt = 1 22 u ) du = 2 du ( u - 6u + 8 t - 3 2t - 4 + 2 2 u + 2 - 3u + 2
Step 4 This integral requires partial fractions to evaluate. However, we first need to do long division on the integrand since the degree of the numerator (3) is higher than the degree of the denominator (2). This gives,

u3 28u - 48 = u +6+ 2 u - 6u + 8 ( u - 2 )( u - 4 )
The form of the partial fraction decomposition on the third term is,

28u - 48 A B = + ( u - 2 )( u - 4 ) u - 2 u - 4
Setting the coefficients equal gives,

28u - 48 = A ( u - 4 ) + B ( u - 2 )
Using the trick to get the coefficients gives,

u = 4 : 64 = 2 B u = 2:
The integral is then,

8 = -2 A

A = -4 B = 32

2007 Paul Dawkins

65

http://tutorial.math.lamar.edu/terms.aspx

Calculus II

u3 4 32 1 du = u + 6 + du = u 2 + 6u - 4 ln u - 2 + 32 ln u - 4 + c 2 u-2 u-4 2 u - 6u + 8
Step 5 The last step is to now do all the back substitutions to get the final answer.

u3 du = t - 2 + 6 2t - 4 - 4ln 2t - 4 - 2 + 32ln 2t - 4 - 4 + c 2 u - 6u + 8

Integrals Involving Quadratics


1. Evaluate the integral

7 dw . w + 3w + 3
2

Step 1 The first thing to do is to complete the square (well leave it to you to verify the completing the square details) on the quadratic in the denominator.

7 7 dw = dw 2 2 w + 3w + 3 (w + 3 ) + 3 2 4
Step 2 From this we can see that the following substitution should work for us.

u = w+ 3 2
Doing the substitution gives,

du = dw

7 7 dw = 2 3 du 2 w + 3w + 3 u +4
Step 3 This integral can be done with the formula given at the start of this section.

7 14 14 2u 2w + 3 dw = tan -1 tan -1 2 +c = +c w + 3w + 3 3 3 3 3
Dont forget to back substitute in for u!

2007 Paul Dawkins

66

http://tutorial.math.lamar.edu/terms.aspx

Calculus II

2. Evaluate the integral

10 x dx . 4 x - 8x + 9
2

Step 1 The first thing to do is to complete the square (well leave it to you to verify the completing the square details) on the quadratic in the denominator.

10 x 10 x dx = dx 2 2 4x - 8x + 9 4 ( x - 1) + 5
Step 2 From this we can see that the following substitution should work for us.

u = x -1
Doing the substitution gives,

du = dx

&

x = u +1

10 x 10 ( u + 1) dx = du 2 2 4x - 8x + 9 4u + 5
Step 3 We can quickly do this integral if we split it up as follows,

10 x 10u 10 10u 5 1 dx = 2 du + 2 du = 2 du + 2 5 du 2 4x - 8x + 9 4u + 5 4u + 5 4u + 5 2 u + 4
After a quick rewrite of the second integral we can see that we can do the first with the substitution v = 4u 2 + 5 and the second is an inverse trig integral we can evaluate using the formula given at the start of the notes for this section.

10 x 5 5 2 -1 2u dx = ln v + tan 2 +c 4x - 8x + 9 4 2 5 5 5 2u = ln 4u 2 + 5 + 5 tan -1 +c 4 5 = 5 2x - 2 2 ln 4 ( x - 1) + 5 + 5 tan -1 +c 4 5

Dont forget to back substitute in for u!

2007 Paul Dawkins

67

http://tutorial.math.lamar.edu/terms.aspx

Calculus II

3. Evaluate the integral

2t + 9
5 2

( t - 14t + 46 )

dt .

Step 1 The first thing to do is to complete the square (well leave it to you to verify the completing the square details) on the quadratic in the denominator.

2t + 9 5 dt = 2 2 ( t - 14t + 46 )

( ( t - 7 ) - 3)
2

2t + 9

5 2

dt

Step 2 From this we can see that the following substitution should work for us.

u =t -7
Doing the substitution gives,

du = dt

&

t =u+7

2 (u + 7 ) + 9 2u + 23 2t + 9 du = 5 dt = 5 5 du 2 2 2 2 u 2 - 3) 2 ( t - 14t + 46 ) ( u - 3) (
Step 3 Next well need to split the integral up as follows,

2t + 9 2u 23 5 dt = 5 du + 5 du 2 2 2 2 2 2 ( t - 14t + 46 ) ( u - 3) ( u - 3)
The first integral can be done with the substitution v = u 2 - 3 and the second integral will require the trig substitution u = 3 sec q . Here is the substitution work.

2t + 9 23 -5 2 dv + 5 dt = v 5 2 2 2 ( t - 14t + 46 ) ( 3sec 2 q - 3)
= v = v
-5 2

3 sec q tan q dq

23 3 sec q tan q dv + dq 5 3tan 2 q ) 2 (

23sec q dv + dq 9 tan 4 q 23 cos3 q -5 dq = v 2 dv + 9 sin 4 q


-5 2

2007 Paul Dawkins

68

http://tutorial.math.lamar.edu/terms.aspx

Calculus II

Now, for the second integral, dont forget the manipulations we often need to do so we can do these kinds of integrals. If you need some practice on these kinds of integrals go back to the practice problems for the second section of this chapter and work some of them. Here is the rest of the integration process for this problem.

2t + 9 23 1 - sin 2 q -5 dt = v 2 dv + cos q dq 5 2 2 9 sin 4 q ( t - 14t + 46 ) = v


-5 2

w = sin q

dv + 23 w-4 - w-2 dw 9
-3 -1 + 23 - 1 ( sin q ) + ( sin q ) + c 9 3

=-2v 3

-3 2

Step 4 We now need to do quite a bit of back substitution to get the answer back into ts. Lets start with the result of the second integration. Converting the q s back to us will require a quick right triangle. From the substitution we have,

sec q =

u hyp = 3 adj

From the right triangle we get,

sin q =

u2 - 3 u

Plugging this into the integral above gives,

2t + 9 2 23u 3 23u dt = +c 5 3 3 + 2 2 2 2 2 2 9 u2 - 3 3 ( u - 3) 27 ( u - 3) ( t - 14t + 46 )


Note that we also back substituted for the v in the first term as well and rewrote the first term a little. Finally, all we need to do is back substitute for the u.

2t + 9 2 5 dt = 2 2 2 ( t - 14t + 46 ) 3 (t - 7) - 3

3 2

27 ( t - 7 ) - 3
2 3

23 ( t - 7 )

3 2

+ 9

23 ( t - 7 )

(t - 7)

-3

+c

= 9

23 ( t - 7 )

(t - 7)

-3

18 + 23 ( t - 7 ) 27 ( t - 7 ) - 3
2

3 2

+c

2007 Paul Dawkins

69

http://tutorial.math.lamar.edu/terms.aspx

Calculus II

Well leave this solution with a final note about these kinds of problems. They are often very long, messy and there are ample opportunities for mistakes so be careful with these and dont get into too much of a hurry when working them.

4. Evaluate the integral

(1 - 4 z - 2 z

3z

2 2

dz .

Step 1 The first thing to do is to complete the square (well leave it to you to verify the completing the square details) on the quadratic in the denominator.

3z 3z dz = 2 2 2 (1 - 4 z - 2 z ) 3 - 2 ( z + 1)

dz

Step 2 From this we can see that the following substitution should work for us.

u = z +1
Doing the substitution gives,

du = dz

&

z = u -1

3 ( u - 1) 3u - 3 3z dz = du = du 2 2 3 - 2u 2 2 2 2 (1 - 4 z - 2 z ) ( 3 - 2u ) ) (
Step 3 Next well need to split the integral up as follows,

3z 3u 3 dz = du - du 2 2 2 2 2 2 (1 - 4 z - 2 z ) ( 3 - 2u ) ( 3 - 2u )
The first integral can be done with the substitution v = 3 - 2u 2 and the second integral will require the trig substitution u =
3 2

sin q . Here is the substitution work.

2007 Paul Dawkins

70

http://tutorial.math.lamar.edu/terms.aspx

Calculus II

3z 3 3 dz = - v -2 dv - 2 2 2 2 4 (1 - 4 z - 2 z ) ( 3 - 3sin q ) = 3 -2 3 v dv - 2 2 4 ( 3cos q )
1 6

(
3 2

3 2

cos q dq

cos q dq

= - 3 v -2 dv 4

sec q dq
3

The second integral for this problem comes down to an integral that was done in the notes for the second section of this chapter and so well just use the formula derived in that section to do this integral. Here is the rest of the integration process for this problem.

3z 3 1 dz = v -1 2 sec q tan q + ln sec q + tan q + c 2 4 2 6 (1 - 4 z - 2 z )


Step 4 We now need to do quite a bit of back substitution to get the answer back into zs. Lets start with the result of the second integration. Converting the q s back to us will require a quick right triangle. From the substitution we have,

sin q =

2 u opp = 3 hyp

From the right triangle we get,

tan q =

2u 3 - 2u
2

&

sec q =

3 3 - 2u 2

Plugging this into the integral above gives,

3z 3 1 6u dz = + ln 2 2 2 4 ( 3 - 2u 2 ) 2 6 3 - 2u (1 - 4 z - 2 z )

3+ 2u +c 3 - 2u 2

Note that we also back substituted for the v in the first term as well and rewrote the first term a little. Finally, all we need to do is back substitute for the u.

3z 3 z +1 1 dz = ln 2 2 2 6 - 4 ( z + 1) 2 6 4 3 - 2 ( z + 1) 1 - 4z - 2z 2 ) (

3 + 2 ( z + 1) 3 - 2 ( z + 1)
2

+c

2007 Paul Dawkins

71

http://tutorial.math.lamar.edu/terms.aspx

Calculus II

Well leave this solution with a final note about these kinds of problems. They are often very long, messy and there are ample opportunities for mistakes so be careful with these and dont get into too much of a hurry when working them.

Integration Strategy
Problems have not yet been written for this section. I was finding it very difficult to come up with a good mix of new problems and decided my time was better spent writing problems for later sections rather than trying to come up with a sufficient number of problems for what is essentially a review section. I intend to come back at a later date when I have more time to devote to this section and add problems then.

Improper Integrals
Problems have not yet been written for this section.

Comparison Test for Improper Integrals


Problems have not yet been written for this section.

Approximating Definite Integrals


Problems have not yet been written for this section.

Applications of Integrals
Introduction
Here are a set of problems for which no solutions are available. The main intent of these problems is to have a set of problems available for any instructors who are looking for some extra problems.
2007 Paul Dawkins 72 http://tutorial.math.lamar.edu/terms.aspx

Calculus II

Note that some sections will have more problems than others and some will have more or less of a variety of problems. Most sections should have a range of difficulty levels in the problems although this will vary from section to section. Here is a list of topics in this chapter that have problems written for them. Arc Length No problems written yet. Surface Area No problems written yet. Center of Mass No problems written yet. Hydrostatic Pressure and Force No problems written yet. Probability No problems written yet.

Arc Length
Problems have not yet been written for this section.

Surface Area
Problems have not yet been written for this section.

Center of Mass
Problems have not yet been written for this section.

Hydrostatic Pressure and Force


Problems have not yet been written for this section.

Probability
Problems have not yet been written for this section.

Parametric Equations and Polar Coordinates


2007 Paul Dawkins 73 http://tutorial.math.lamar.edu/terms.aspx

Calculus II

Introduction
Here are a set of problems for which no solutions are available. The main intent of these problems is to have a set of problems available for any instructors who are looking for some extra problems. Note that some sections will have more problems than others and some will have more or less of a variety of problems. Most sections should have a range of difficulty levels in the problems although this will vary from section to section. Here is a list of topics in this chapter that have problems written for them. Parametric Equations and Curves No problems written yet. Tangents with Parametric Equations No problems written yet. Area with Parametric Equations No problems written yet. Arc Length with Parametric Equations No problems written yet. Surface Area with Parametric Equations No problems written yet. Polar Coordinates No problems written yet. Tangents with Polar Coordinates No problems written yet. Area with Polar Coordinates No problems written yet. Arc Length with Polar Coordinates No problems written yet. Surface Area with Polar Coordinates No problems written yet. Arc Length and Surface Area Revisited No problems written yet.

Parametric Equations and Curves


Problems have not yet been written for this section.

Tangents with Parametric Equations


Problems have not yet been written for this section.

Area with Parametric Equations


Problems have not yet been written for this section.

Arc Length with Parametric Equations


Problems have not yet been written for this section.
2007 Paul Dawkins 74 http://tutorial.math.lamar.edu/terms.aspx

Calculus II

Surface Area with Parametric Equations


Problems have not yet been written for this section.

Polar Coordinates
Problems have not yet been written for this section.

Tangents with Polar Coordinates


Problems have not yet been written for this section.

Area with Polar Coordinates


Problems have not yet been written for this section.

Arc Length with Polar Coordinates


Problems have not yet been written for this section.

Surface Area with Polar Coordinates


Problems have not yet been written for this section.

Arc Length and Surface Area Revisited


Problems have not yet been written for this section and probably wont be to be honest since this is just a summary section.

2007 Paul Dawkins

75

http://tutorial.math.lamar.edu/terms.aspx

Calculus II

Sequences and Series


Introduction
Here are a set of problems for which no solutions are available. The main intent of these problems is to have a set of problems available for any instructors who are looking for some extra problems. Note that some sections will have more problems than others and some will have more or less of a variety of problems. Most sections should have a range of difficulty levels in the problems although this will vary from section to section. Here is a list of topics in this chapter that have problems written for them. Sequences No problems written yet. More on Sequences No problems written yet. Series The Basics No problems written yet. Series Convergence/Divergence No problems written yet. Series Special Series No problems written yet. Integral Test No problems written yet. Comparison Test/Limit Comparison Test No problems written yet. Alternating Series Test No problems written yet. Absolute Convergence No problems written yet. Ratio Test No problems written yet. Root Test No problems written yet. Strategy for Series No problems written yet. Estimating the Value of a Series No problems written yet. Power Series No problems written yet. Power Series and Functions No problems written yet. Taylor Series No problems written yet. Applications of Series No problems written yet. Binomial Series No problems written yet.

Sequences
Problems have not yet been written for this section.

More on Sequences
Problems have not yet been written for this section.

2007 Paul Dawkins

76

http://tutorial.math.lamar.edu/terms.aspx

Calculus II

Series The Basics


Problems have not yet been written for this section.

Series Convergence/Divergence
Problems have not yet been written for this section.

Series Special Series


Problems have not yet been written for this section.

Integral Test
Problems have not yet been written for this section.

Comparison Test / Limit Comparison Test


Problems have not yet been written for this section.

Alternating Series Test


Problems have not yet been written for this section.

Absolute Convergence
Problems have not yet been written for this section.

Ratio Test
Problems have not yet been written for this section.
2007 Paul Dawkins 77 http://tutorial.math.lamar.edu/terms.aspx

Calculus II

Root Test
Problems have not yet been written for this section.

Strategy for Series


Problems have not yet been written for this section.

Estimating the Value of a Series


Problems have not yet been written for this section.

Power Series
Problems have not yet been written for this section.

Power Series and Functions


Problems have not yet been written for this section.

Taylor Series
Problems have not yet been written for this section.

Applications of Series
Problems have not yet been written for this section.

Binomial Series

2007 Paul Dawkins

78

http://tutorial.math.lamar.edu/terms.aspx

Calculus II

Problems have not yet been written for this section.

Vectors
Introduction
Here are a set of problems for which no solutions are available. The main intent of these problems is to have a set of problems available for any instructors who are looking for some extra problems. Note that some sections will have more problems than others and some will have more or less of a variety of problems. Most sections should have a range of difficulty levels in the problems although this will vary from section to section. Here is a list of topics in this chapter that have problems written for them. Vectors The Basics No problems written yet. Vector Arithmetic No problems written yet. Dot Product No problems written yet. Cross Product No problems written yet.

Vectors The Basics


Problems have not yet been written for this section.

Vector Arithmetic
Problems have not yet been written for this section.

Dot Product
Problems have not yet been written for this section.

Cross Product
Problems have not yet been written for this section.
2007 Paul Dawkins 79 http://tutorial.math.lamar.edu/terms.aspx

Calculus II

Three Dimensional Space


Introduction
Here are a set of problems for which no solutions are available. The main intent of these problems is to have a set of problems available for any instructors who are looking for some extra problems. Note that some sections will have more problems than others and some will have more or less of a variety of problems. Most sections should have a range of difficulty levels in the problems although this will vary from section to section. Here is a list of topics in this chapter that have problems written for them. The 3-D Coordinate System No problems written yet. Equations of Lines No problems written yet. Equations of Planes No problems written yet. Quadric Surfaces No problems written yet. Functions of Several Variables No problems written yet. Vector Functions No problems written yet. Calculus with Vector Functions No problems written yet. Tangent, Normal and Binormal Vectors No problems written yet. Arc Length with Vector Functions No problems written yet. Curvature No problems written yet. Velocity and Acceleration No problems written yet. Cylindrical Coordinates No problems written yet. Spherical Coordinates No problems written yet

The 3-D Coordinate System


Problems have not yet been written for this section.

Equations of Lines
Problems have not yet been written for this section.

Equations of Planes

2007 Paul Dawkins

80

http://tutorial.math.lamar.edu/terms.aspx

Calculus II

Problems have not yet been written for this section.

Quadric Surfaces
Problems have not yet been written for this section.

Functions of Several Variables


Problems have not yet been written for this section.

Vector Functions
Problems have not yet been written for this section.

Calculus with Vector Functions


Problems have not yet been written for this section.

Tangent, Normal and Binormal Vectors


Problems have not yet been written for this section.

Arc Length with Vector Functions


Problems have not yet been written for this section.

Curvature
Problems have not yet been written for this section.

2007 Paul Dawkins

81

http://tutorial.math.lamar.edu/terms.aspx

Calculus II

Velocity and Acceleration


Problems have not yet been written for this section.

Cylindrical Coordinates
Problems have not yet been written for this section.

Spherical Coordinates
Problems have not yet been written for this section.

2007 Paul Dawkins

82

http://tutorial.math.lamar.edu/terms.aspx

Potrebbero piacerti anche